cart-icon Товаров: 0 Сумма: 0 руб.
г. Нижний Тагил
ул. Карла Маркса, 44
8 (902) 500-55-04

Тест по физике 7 класс скорость единицы скорости ответы: Тест по физике Скорость. Единицы скорости с ответами

Содержание

Тест по физике Скорость. Единицы скорости с ответами

Тесты по физике 7 класс. Тема: «Скорость. Единицы скорости»

Правильный вариант ответа отмечен знаком +

1. Скорость — это физическая величина, которая показывает…

+ какой путь проходит тело за единицу времени

— сколько тело потратило сил

— расстояние, пройденное телом

— время, потраченное телом

2. В СИ скорость обозначается…

+ м/с

— м

— км/ч

— с

3. Формула нахождения скорости:

+ v=s/t

v = s*t

v = s+t

v = s-t

4. Выберите верное утверждение:

+ Чем больше скорость, тем больший путь проходит тело за единицу времени.

— Чем больше скорость, тем меньший путь проходит тело за единицу времени.

— Чем больше скорость, тем больше время, затраченное на путь.

— Чем больше скорость, тем сложнее ее измерить.

5. Как называется прибор для измерения скорости?

+ спидометр

— барометр

— манометр

— динамометр

6. Определите скорость движения автомобиля (в км/ч), если за 4 часа пути он проехал 340 км.

+85 км/ч

— 80 км/ч

— 95 км/ч

— 90 км/ч

7. Расстояние между Москвой и Новосибирском 3200 км. Поезд проходит этот путь за 64 ч. С какой скоростью он двигается?

+ 50 км/ч

— 40 км/ч

— 60 км/ч

— 55 км/ч

8. Скорость самолета, равную 810 км/ч, выразите в СИ.

+ 225 м/с

— 210 м/с

— 270 м/с

— 245 м/с

9. Велосипедист проехал за 30 мин путь равный 7200 м. Вычислите среднюю скорость движения велосипедиста (в м/с).

+ 4 м/с

— 120 м/с

— 24 м/с

— 240 м/с

тест 10. По шоссе навстречу друг другу равномерно и прямолинейно движутся легковой автомобиль со скоростью v1=72 км/ч и трактор со скоростью v2=10 м/с. Определите расстояние между автомобилем и трактором через время t = 20 мин после их встречи.

+ 36 км

— 62 км

— 42 км

— 34 км

11. Через сколько минут ракета, запущенная с поверхности Земли со скоростью 8 км/с, отлетит от нее на расстояние 480 км?

+ 1 мин

— 0,5 мин

— 1,5 мин

— 2 мин

12.. Определите во сколько раз отличаются скорости движения лыжника (Л) и пешехода (П), графики пути которых предоставлены на рисунке 1.

+ 3

-2

-6

-5

13. С Земли на Луну послан радиосигнал. Спустя время t = 2,6 с после посылки отраженный о лунной поверхности сигнал принят на Земле. Рассчитайте расстояние от Земли до Луны. Скорость радиоволн v = 300 000 км/с.

+ 390 000 км

— 180 000 км

— 285 000 км

— 420 000 км

14. Пусть 40 км голубь пролетает за 50 мин. Определите путь, который он пролетел за 15 минут.

+ 12 км

— 10 км

— 15 км

— 17 км

15. По дороге равномерно движутся два велосипедиста. Один из них, имеющий скорость 8 м/с, проехал некоторый путь за 15 с. Второй велосипедист проехал тот же путь за 10 с. Определите скорость движения второго велосипедиста.

+ 12 м/с

— 15 м/с

— 9 м/с

— 16 м/с

16. За какое время поезд, движущийся равномерно со скоростью 36 км/ч, пройдет тоннель длиной 160 м, если длина поезда 80 м?

+ 24 с

— 60 с

— 120 с

— 36 с

17. Сколько времени из окна электропоезда, идущего со скоростью 72 км/ч, пассажир будет видеть встречный поезд длиной 250 м, скорость которого 36 км/ч?

+ 8,3 с

— 5,8 с

— 6,9 с

— 9,2 с

18. Две лодки начали одновременно двигаться по реке в противоположных от берега направлениях. Скорость лодки, плывущей по течению, 4 м/с, а скорость лодки, плывущей против течения, 2 м/с. Каким будет расстояние между лодками через 10 минут?

+ 3,6 км

— 4,2 км

— 4 км

— 3,3 км

19. Гончая собака пробегает путь равный 200 м с постоянной скоростью 25 м/с. Сколько времени понадобилось бы щуке, чтобы преодолеть такой же путь, если бы она плыла со скоростью 2 м/с?

+ 1,7 мин

— 1,5 мин

— 1,8 мин

— 1,4 мин

тест-20. Самолет, набрав высоту, летит с постоянной скоростью и за время 10 мин пролетел 140 км. Какой путь пролетит самолет за 45 минут, двигаясь с той же скоростью?

+ 630 км

— 450 км

— 580 км

— 740 км

21. По графику зависимости скорости движения тела от времени, приведенному на рисунке, определите путь, пройденный телом за промежуток времени 8 с.

+ 1,6 м

— 8 м

— 0,2 м

— 2,4 м

22. Путь равный 1,8 км от дома до парка спортсмен пробежал со скоростью 4 м/с, а обратный путь прошел быстрым шагом со скоростью 2 м/с. Какова средняя скорость движения спортсмена?

+ 2,7 м/с

— 3 м/с

— 2,5 м/с

— 3,2 м/с

23. Автомобиль и мопед равномерно движутся в одном направлении по параллельным полосам прямолинейного участка шоссе. Скорость движения автомобиля 90 км/ч, мопеда – 20 км/ч. Каким будет расстояние между ними через 3 минуты, если в начальный момент они находились рядом?

+ 900 м

— 1 км

— 1,2 км

— 800 м

24. Из поселка в направлении озера вышел пешеход и двигался по прямой дороге с постоянной скоростью 5 км/ч. Через 30 минут из того же поселка начал равномерное движение в том же направлении велосипедист со скоростью 5 м/с и догнал пешехода у озера. Через какой промежуток времени после начала своего движения велосипедист догнал пешехода?

+ 12 минут

— 15 минут

— 10 минут

— 14 минут

25. Из двух населенных пунктов навстречу друг другу выехали два мотоциклиста. Скорость первого мотоциклиста 100 км/ч, а скорость второго — 120 км/ч. Через 2 часа они встретились. Определите расстояние между населенными пунктами.

+ 440 км

— 220 км

— 480 км

— 360 км

26. Моторная лодка за 3 ч проходит расстояние от города до поселка, расположенного ниже по течению реки. Сколько времени займет обратный путь, если скорость движения лодки относительно воды в 4 раза больше скорости течения?

+ 5 ч

— 4 ч

— 3 ч

— 6 ч

27. Земноводный танк может двигаться на гусеницах по суше со скоростью 70 км/ч и плавать со скоростью 10 км/ч. Сколько времени потребуется этому танку, чтобы пройти общее расстояние 61 км, если на пути будет озеро шириной 5 км?

+ 1,3 ч

— 1,5 ч

— 1,1 ч

— 1,4 ч

28. В течение 2 часов электричка двигалась со скоростью 110 км/ч, затем сделала остановку на 10 мин. Оставшуюся часть пути она шла со скоростью 90 км/ч. Какова средняя скорость электрички на всем пути, если она прошла 400 км?

+ 96 км/ч

— 100 км/ч

— 67 км/ч

— 91 км/ч

29. Поезд длиной 150м движется по мосту равномерно со скоростью 36км/ч. За сколько минут он пройдет мост, если его длина 750 м?

+ 1,5 мин

— 5 мин

— 4,1 мин

— 2,4 мин

тест_30. Человек стреляет из пистолета в мишень, находящуюся от него на расстоянии 33 м. Через какое время после выстрела, он услышит звук от удара пули в мишень, если скорость пули равна 660 м/с, а скорость распространения звука -330 м/с?

+ 0,15с

-1 с

— 0,3 с

— 0,8 с

Тесты по физике Скорость (7 класс)

Последний раз тест пройден 24 часа назад.

Для учителя

  1. Вопрос 1 из 10

    Скорость — это физическая величина, которая показывает

    • пройденный телом путь

    • как быстро движется тело

    • какой путь проходит тело за единицу времени

    Подсказка

    Правильный ответ

    Неправильный ответ

    В вопросе ошибка?

  2. Вопрос 2 из 10

    Какая принята единица скорости в СИ?

    • Миллиметр в секунду (мм/с)

    • Сантиметр в секунду (см/с)

    • Метр в секунду (м/с)

    • Километр в час (км/ч)

    Подсказка

    Правильный ответ

    Неправильный ответ

    В вопросе ошибка?

  3. Вопрос 3 из 10

    Найдите скорость (в м/с) равномерного в течение 1,5 мин по­лёта воздушного шара, за которые он пролетел 540 м

    • 15 м/с

    • 6 м/с

    • 54 м/с

    • 10 м/с

    Подсказка

    Правильный ответ

    Неправильный ответ

    В вопросе ошибка?

  4. Вопрос 4 из 10

    Какова скорость пешехода, преодолевающего ровным шагом расстояние 1,2 км за 20 мин?

    • 2 м/с

    • 2 км/ч

    • 1 м/с

    • 10 м/с

    Подсказка

    Правильный ответ

    Неправильный ответ

    В вопросе ошибка?

  5. Вопрос 5 из 10

    Определите скорость (в м/с) равномерного скольжения конь­кобежца, который за 5 мин проехал путь длиной 1,5 км

    • 5 м/с

    • 3 м/с

    • 300 м/с

    • 30 м/с

    Подсказка

    Правильный ответ

    Неправильный ответ

    В вопросе ошибка?

  6. Вопрос 6 из 10

    Рассчитайте среднюю скорость движения (в м/с) поезда между двумя станциями, расстояние между которыми 30 км, если он, выйдя с одной станции, прибыл на другую через 30 мин

    • 1 м/с

    • 10 м/с

    • ≈ 167 м/с

    • ≈ 16,7 м/с

    Подсказка

    Правильный ответ

    Неправильный ответ

    В вопросе ошибка?

  7. Вопрос 7 из 10

    Определите среднюю скорость автомобиля на пути длиной 300 км.

    Первую половину этого пути он прошёл за 1,5 ч, а вто­рую за 2,5 ч
    • 80 км/ч

    • 40 км/ч

    • 60 км/ч

    • 75 км/ч

    Подсказка

    Правильный ответ

    Неправильный ответ

    В вопросе ошибка?

  8. Вопрос 8 из 10

    Одна из моделей европейского автомобиля «Феррари» дости­гает скорости 360 км/ч. Какой путь пройдёт этот автомобиль за 10 мин с такой скоростью?

    • 10 км

    • 60 км

    • 36 км

    • 600 км

    Подсказка

    Правильный ответ

    Неправильный ответ

    В вопросе ошибка?

  9. Вопрос 9 из 10

    Самый быстрый пока серийный американский автомобиль разгоняется примерно до 430 км/ч. Считая его скорость рав­ной 400 км/ч, рассчитайте, сколько времени ему понадобится, чтобы проехать 40 км

    • 6 мин

    • 10 ми

    • ≈ 17 мин

    • 5 мин

    Подсказка

    Правильный ответ

    Неправильный ответ

    В вопросе ошибка?

  10. Вопрос 10 из 10

    Автобус проехал свой маршрут от станции до конечной оста­новки за 0,8 ч со средней скоростью 32 км/ч.

    Чему равен его путь?
    • 32 км

    • 25,6 км

    • 40 км

    • 20 км

    Подсказка

    Правильный ответ

    Неправильный ответ

    В вопросе ошибка?

Доска почёта

Чтобы попасть сюда — пройдите тест.

    Пока никого нет. Будьте первым!

Тесты по физике «Скорость» разработаны для учащихся средней школы. Они помогут закрепить знания, выявить плохо усвоенный материал. Выполнение теста требует знания терминологии и единиц скорости в системе СИ. Более сложные задания представляют собой задачи по теме. Уже поданные правильные ответы позволят сразу запоминать то, что «упущено». Вопросы охватывают информацию, которая предусмотрена программой и является обязательной для изучения.

Тест «Скорость» (7 класс) – один из самых удобных способов подготовки к уроку или проверочной работе.

Рейтинг теста

3. 8

Средняя оценка: 3.8

Всего получено оценок: 421.


А какую оценку получите вы? Чтобы узнать — пройдите тест.

тесты по физике 7 класс Чеботарева. К учебнику Перышкина | Ответы

Тест по физике тесты 7

В 7 классе учащихся ждет увлекательный экскурс, позволяющий постичь окружающий мир и получить массу интересных сведений в курсе физики:
1. Каково первоначальное строение вещества?
2. Как взаимодействуют тела?
3. Что представляют собой давление, мощность и энергия?

Без тщательной проработки полученной на уроке информации, знания улетучатся. Чтобы этого не произошло, рекомендуется использовать тесты по физике А. В. Чеботарева, а проверить правильность выполнения заданий помогут . Ответственные родители часто делают домашние задания вместе с детьми, однако многие уже успели подзабыть школьную программу, и подготовка уроку может затянуться, а КПД снизиться. Чтобы исключить неловкие моменты, используйте учебник А. В. Перышкина, тематические тесты для самоконтроля и ! Так систематические тренировки пройдут быстрее и продуктивнее.

Решебник придет на выручку каждому, кто не стремится просто списать ответы. Задача проста: пройти тест и сверится с мнением экспертов. Этот подход признан наиболее эффективным. Не получается? Рекомендуется повторить нужный параграф и попробовать еще раз. Обычно достаточно всего нескольких попыток.

Классы:
Предметы:
Помощь с учёбой:
Наша компания:

© 2021 Copyright. Все права защищены. Правообладатель SIA Ksenokss.
Адрес: 1069, Курземес проспект 106/45, Рига, Латвия.
Тел.: +371 29-851-888 E-mail: [email protected]

Наша компания.

Www. euroki. org

16.05.2020 10:21:19

2020-05-16 10:21:19

Источники:

Https://www. euroki. org/gdz/ru/fizika/7_klass/testy-po-fizike-7-klass-fgos-chebotareva-k-uchebniku-peryshkina-25

Тесты по физике для 7 класса онлайн | Online Test Pad » /> » /> .keyword { color: red; }

Тест по физике тесты 7

Тестирование учащихся 7 классов средней школы по физике. Виды сил. Сила тяжести, упругости, трения, вес тела.

ВПР-2022. Физика, 7 класс

Тест предназначен для подготовки к ВПР по физике в 7 классе. Он составлен по демоверсии ВПР 2022 года

Механическое движение

Тематический тест для учеников 7 класса. Предназначен для проверки и закрепления параграфов 13-16 учебника А. В. Перышкина «Физика-7»

Физика 7 класс. Скорость. Единицы скорости.(2)

В тесте необходимо перевести скорость из одной единицы измерения в другую. Показаны примеры перевода. По окончании Вы можете распечатать сертификат об итогах прохождения теста.

Инерция. Взаимодействие тел. Масса тала. Плотность вещества

Тест по физике 7 класса по теме «Инерция. Взаимодействие тел. Масса тела. Плотность вещества

«Механическая работа и мощность». Физика 7 класс

Тест предназначен для учащихся 7 класса для проверки знаний по теме «Механическая работа. Мощность»

Физические величины.

Обозначения, единицы измерения и перевод в систему СИ

Тест предназначен для отработки знаний по обозначению физических величин, единиц измерения и перевода в систему СИ.

Давление твердых тел, жидкостей и газов

Тест по теме «Давление» содержит 10 заданий с выбором правильного ответа. Предназначен для учащихся 7 класса для проверки знаний после прохождения темы. Будет полезен идля учащихся 9 класса для подготовки к ГИА по физике.

Масса тела

Данный тест предназначен для обучающихся 7 класса для проверки усвоения физического понятия «масса»

Молекулы. Движение молекул.

Первоначальные сведения о строении вещества. Тест Молекулы. Движение молекул.

Тест по теме «Сила Архимеда» 7 класс

Тест по теме «Сила Архимеда. Условия плавания тел», проверяет уровень усвоения пройденного материала

Физика 7 класс Введение

Тестирование учащихся 7 классов средней школы по физике. Тема: Введение. Тесты с выбором вариантов. Просмотр ответов.

ВПР 2019 по физике для 7 класса

Образец всероссийской проверочной работы для 7 класса по физике 2019 года

ВПР по физике — 7 класс. Тренировочная работа № 1

ВПР-2022, физика для учащихся 7-х классов общеобразовательных школ.

Физика-7. Сила трения

Вашему вниманию будет предложен тест из 16-ти вопросов. Внимательно читайте каждый вопрос, потому что иногда возникает ощущение, что вам дают одинаковые вопросы. Это не совсем так: просто одно и то же явление рассматривается с разных точек зрения. В этом тесте не предусмотрены подсказки, но посмотреть правильные ответы после написания теста, Вы сможете.

Физика 7 класс Строение вещества

Тестирование учащихся 7 классов средней школы по физике. Тема: Строение вещества. Тесты с выбором вариантов. Просмотр ответов.

Тест по физике-7. Простые механизмы, работа, мощность

Тест по физике для 7 класса проверяет знания обучающихся по вопросам работы, мощности, простых механизмов. Желаю удачи!

Плотность 7 класс

Тест на тему:»Плотность вещества» для учащихся 7-х классов по предмету физика

Давление

Для закрепления тем «Давление. Единицы давления. Способы увеличения и уменьшения давления».

Тест по теме: «Давление газов, жидкостей и твёрдых тел»

Тест по основным понятиям темы: «Давление газов, жидкостей и твёрдых тел»

Тест «Плавание тел»

Физика 7 класс. Тема «Сила Архимеда. Плавание тел». Тест «Плавание тел» адресован учащимся 7 класса для самопроверки знаний по физике. Тест состоит из 16 заданий. В каждом задании необходимо выбрать один правильный ответ из предложенных, записать ответ словом или цифрой, собрать слово, записать ответ в виде слова или цифры. Тест иллюстрирован.

Тест по теме «Физические величины» 7 класс

Тест по теме «Физические величины» выполняют учащиеся 7 классов в начале учебного года, после изучения темы «Введение». Тест дается с целью проверки прочности усвоения учащимися знаний о физических понятиях и явлениях, о названиях физических величин, их обозначении и единицах измерения в международной системе счисления СИ.

Школьная олимпиада по физике для учащихся 7 класса

Тест по физике для учащихся 7 класса для проведения школьной олимпиады

Давление твердых тел

Тест по физике для учащихся 7 класса по теме «Давление твердых тел»

Олимпиада по физике 7 класс

Этот тест сделан для проверки ваших знаний! Сертификат выдаётся. Делаем вид, что мы делаем тест для олимпиады! Думайте, решайте и радуйтесь! Удачи каждому)))

Физика 7 класс. Силы.

Тест «Силы» может быть испоьзован для контроля знаний, закрепления материала послеизучения соответствующего материала по данной теме в 7 классе или внеклассных мероприятиях. Содержит десять вопросов по данной теме.

Плотность

Тест на закрепление темы «Плотность». Задачи разной степени сложности. Требуется расчет.

Давление жидкостей

Тест применяется при повторении и закреплении темы «Давление жидкостей».

Давление твердых тел, жидких и газообразных.

Физика 7 класс.

Данный тест предполагает проверку качества усвоения учащимися материала по теме «Давление твёрдых тел, жидкостей и газов»

Физика-7Т. Механическое движение. Вариант 1Т

Вашему вниманию будут предложены 4 теста по этой теме: два тренировочные (рекомендуем начинать выполнение именно с них), два другие теста — контрольные. ЭТОТ тест является ТРЕНИРОВОЧНЫМ. В тренировочных тестах предусмотрены комментарии-подсказки, которые можно посмотреть ДО ответа на вопрос. В контрольных тестах подсказок нет.

Физика-7К. Механическое движение. Вариант 2К

Вашему вниманию будут предложены 4 теста по этой теме: два тренировочные (рекомендуем начинать выполнение именно с них), два другие теста — контрольные. ЭТОТ тест является КОНТРОЛЬНЫМ. В тренировочных тестах предусмотрены комментарии-подсказки, которые можно посмотреть ДО ответа на вопрос. В контрольных тестах подсказок нет.

Физика-7Т. Инерция. Вариант 1Т

Вашему вниманию будут предложены 2 теста по этой теме: один тренировочный (рекомендуем начинать выполнение именно с него), другой — контрольный. ЭТОТ тест является ТРЕНИРОВОЧНЫМ. В тренировочных тестах предусмотрены комментарии-подсказки, которые можно посмотреть ДО ответа на вопрос. В контрольных тестах подсказок нет.

Физика-7Т. Взаимодействие тел. Масса тела. Вариант 1Т

Вашему вниманию будут предложены 4 теста по этой теме: два тренировочные (рекомендуем начинать выполнение именно с них), два другие теста — контрольные. ЭТОТ тест является ТРЕНИРОВОЧНЫМ. В тренировочных тестах предусмотрены комментарии-подсказки, которые можно посмотреть ДО ответа на вопрос. В контрольных тестах подсказок нет.

Физика-7. Сила — векторная величина. Сложение сил.

Вашему вниманию будет предложен тест из 12-ти вопросов. Внимательно читайте каждый вопрос, потому что иногда возникает ощущение, что вам дают одинаковые вопросы. Это не совсем так: просто одно и то же явление рассматривается с разных точек зрения. В этом тесте не предусмотрены подсказки, но посмотреть правильные ответы после написания теста, Вы сможете.

Физика. Решение задач на скорость

Завершив этот тест, вы проверите, как вы научились решать задачи на скорость.

Физика. Решение задач на плотность.

Завершив этот тест, вы проверите, как вы научились решать задачи наплотность вещества.

7 класс физика

Данный тест предназначен для определения знания предмета «Физика» за курс7 класса.

«Архимедова сила. Плавание тел». Физика 7 класс

Тест адресован учащимся 7 класса для самопроверки зниний по физике по теме «Архимедова сила. Плавание тел»

Физика 7 класс. Механическая работа. Единицы работы

Тест по теме: Механическая работа. Единицы работы. Для того чтобы выполнить данный тест, Вам необходимо прочитать теорию во вложении, просмотреть видеоурок, затем только выполнять.

Цена деления прибора

Тест для учащихся 7 класса. Используется для проверки знаний правил определения цены деления и показаний прибора.

Механическое движение

Тематический тест для учеников 7 класса. Предназначен для проверки знаний по теме «Механическое движение».

Промежуточная аттестация по физике в 7 классе

Тест рассчитан на учащихся 7 классов, изучающих курс физики на базовом уровне. Тест состоит из 16 вопросов трёх типов (А, В и С). Часть А состоит из 14 вопросов или задач и по одной задаче типа В и С. В задачах части А нужно либо выбрать правильный ответ из предложенных вариантов, либо ввести числовое значение. Каждое верно выполненное задание оценивается 1 баллом. Задача части В — задание на соответствие понятий, терминов, формул. Каждое верное соответствие оценивается одним баллом. Задача части С — вычислительная задача. Верно выполненное задание оценивается 3 баллами. Вариант генерируется случайным образом из базы данных по каждой задаче. Ответ формируется автоматически, но его можно продублировать по e-mail (после прохождения теста есть специальная форма).

Тематический тест для учеников 7 класса. Предназначен для проверки и закрепления параграфов 13-16 учебника А. В. Перышкина «Физика-7»

В тесте необходимо перевести скорость из одной единицы измерения в другую. Показаны примеры перевода. По окончании Вы можете распечатать сертификат об итогах прохождения теста.

Требуется расчет.

Onlinetestpad. com

19.12.2017 8:15:44

2017-12-19 08:15:44

Источники:

Https://onlinetestpad. com/ru/tests/physics/7class

Тесты по Физике для 7 класса » /> » /> .keyword { color: red; }

Тест по физике тесты 7

В данном каталоге представлены интерактивные компьютерные тесты по «Физике» для 7 класса. Любой тест, который находится на нашем портале, можно загрузить и использовать на своем локальном компьютере, либо решать и проверять ответы прямо на сайте

Инструкция по выполнению работы На выполнение работы по физике даётся 40 минут. Работа содержит 11 заданий. Всего можно набрать 17 баллов

В данном каталоге представлены интерактивные компьютерные тесты по Физике для 7 класса.

Testedu. ru

24.05.2018 1:13:05

2018-05-24 01:13:05

Источники:

Https://testedu. ru/test/fizika/7-klass/

Физика: уроки, тесты, задания.

  • Предметы
    1. Введение. Макро- и микромир. Числа со степенью 10
    2. Наблюдения, опыты, измерения, гипотеза, эксперимент
    3. Физические величины.
      Международная система единиц
    1. Строение вещества. Молекулы и атомы
    2. Броуновское движение. Диффузия
    3. Притяжение и отталкивание молекул. Смачивание и капиллярность
    4. Изменение свойств веществ.
      Агрегатные состояния вещества
    1. Механическое движение. Траектория и путь
    2. Скорость. Неравномерное движение. Средняя скорость
    3. Что такое инерция
    4. Взаимодействие тел.
      Масса тела. Измерение массы тела на весах
    5. Плотность вещества. Связь массы, объёма тела с его плотностью
    6. Что такое сила. Сила гравитации. Сила тяжести
    7. Что такое вес тела. Свободное падение
    8. Измерение силы с помощью динамометра
    9. Деформации тел.
      Сила упругости. Закон Гука
    10. Взаимодействие тел. Сила трения
    1. Что такое давление и сила давления
    2. Давление твёрдых тел
    3. Давление газа.
      Применение сжатого воздуха
    4. Атмосферное давление и его измерение. Опыт Торричелли
    5. Давление в жидкости. Закон Паскаля
    6. Гидростатическое давление. Давление на дне морей и океанов
    7. Сообщающиеся сосуды.
      Водопровод. Шлюзы
    8. Гидравлический пресс. Насосы
    9. Закон Архимеда. Вес тела в жидкости
    10. Действие жидкости и газа на погружённое в них тело. Плавание тел
    11. Выталкивающая сила в газах.
      Воздухоплавание
    1. Работа как физическая величина
    2. Мощность как характеристика работы
    3. Простые механизмы. Рычаг. Наклонная плоскость
    4. Подвижные и неподвижные блоки
    5. Полезная работа.
      Коэффициент полезного действия
    6. Энергия как физическая величина. Виды энергии
    1. Температура
    2. Виды теплопередачи
    3. Количество теплоты как физическая величина
    4. Что такое удельная теплоёмкость вещества
    5. Что такое удельная теплота сгорания топлива
    1. Плавление и отвердевание тел.
      Температура плавления
    2. Что такое удельная теплота плавления
    3. Парообразование и конденсация
    4. Относительная влажность воздуха и её измерение. Психрометр
    5. Кипение.
      Температура кипения. Удельная теплота парообразования
    6. Объяснение изменений агрегатных состояний вещества
    7. Преобразования энергии в тепловых машинах
    8. Экологические проблемы использования тепловых машин
    9. Закон сохранения энергии в тепловых процессах
    1. Электризация тел.
      Два рода электрических зарядов
    2. Проводники, диэлектрики и полупроводники
    3. Взаимодействие заряженных тел. Электрическое поле
    4. Закон сохранения электрического заряда
    5. Дискретность электрического заряда.
      Электрон. Строение атомов
    6. Электрический ток. Электрическая цепь. Гальванические элементы
    7. Электрический ток в различных средах. Действия электрического тока
    8. Сила тока как физическая величина. Амперметр
    9. Электрическое напряжение как физическая величина.
      Вольтметр
    10. Электрическое сопротивление как физическая величина. Закон Ома
    11. Удельное сопротивление. Реостаты. Резисторы
    12. Последовательное и параллельное соединения проводников. Правила
    13. Понятия работы и мощности электрического тока
    14. Количество теплоты, выделяемое проводником с током
    15. Счётчик электрической энергии
    16. Виды ламп накаливания
    17. Расчёт электроэнергии, потребляемой бытовыми электроприборами
    18. Короткое замыкание.
      Электробезопасность. Плавкие предохранители
    1. Магнитное поле. Направление магнитных линий
    2. Однородное и неоднородное магнитное поле
    3. Направление магнитных линий прямого проводника с током
    4. Как обнаружить магнитное поле.
      Правило левой руки
    5. Что такое индукция магнитного поля и магнитный поток
    6. Что такое электромагнитная индукция
    7. Направление индукционного тока. Правило Ленца. Явление самоиндукции
    8. Переменный ток.
      Генератор переменного тока
    9. Трансформатор. Передача электрической энергии на расстояние
    10. Свойства электромагнитов
    11. Постоянные магниты. Магнитное поле Земли
    12. Движение проводника в магнитном поле.
      Электродвигатель. Динамик и микрофон
    1. Понятие материальной точки. Системы отсчёта
    2. Перемещение. Скорость прямолинейного равномерного движения
    3. Прямолинейное равноускоренное движение: мгновенная скорость, ускорение
    4. Равнопеременное движение
    5. Графики зависимости величин от времени при равноускоренном движении
    6. Графики зависимости кинематических величин от времени при равномерном и равноускоренном движении
    1. Относительность механического движения
    2. Первый закон Ньютона.
      Инерция. Инерциальные системы отсчёта
    3. Второй закон Ньютона. Сила трения скольжения
    4. Взаимодействие тел. Третий закон Ньютона
    5. Ускорение свободного падения. Изменение веса при движении
    6. Движение тела, брошенного вертикально вверх.
      Невесомость
    7. Закон всемирного тяготения. Гравитационная постоянная
    1. Понятие импульса тела
    2. Закон сохранения импульса. Виды взаимодействий
    3. Что такое реактивное движение
    4. Механическая работа и мощность.
      Закон сохранения механической энергии
    1. Колебательное движение. Амплитуда, частота, период колебаний
    2. Колебательная система. Колебания груза на пружине. Математический маятник
    3. Превращение энергии при колебательном движении
    4. Вынужденные колебания.
      Резонанс
    5. Поперечные и продольные волны. Длина волны
    6. Звуковые волны. Скорость звука
    7. От чего зависят высота, тембр, громкость и резонанс звука
    1. Электромагнитное поле.
      Скорость распространения электромагнитных волн
    2. Конденсатор. Колебательный контур. Принципы радиосвязи и телевидения
    3. Электромагнитная теория света
    1. Источники света.
      Прямолинейность распространения света
    2. Понятие отражения света. Закон отражения. Плоское зеркало
    3. Понятие преломления света. Закон преломления
    4. Линза. Фокусное расстояние линзы. Построение изображений
    5. Оптическая сила линзы.
      Глаз как оптическая система. Оптические приборы
    6. Дисперсия. Спектр. Типы оптических спектров
    1. Радиоактивность как свидетельство сложного строения атомов. Опыты Резерфорда
    2. Протонно-нейтронная модель ядра.
      Энергия связи частиц в ядре
    3. Постулаты Бора. Поглощение и испускание света атомами. Линейчатые спектры
    4. Альфа-, бета- и гамма-излучения. Радиоактивные превращения атомных ядер
    5. Методы наблюдения и регистрации частиц в ядерной физике
    6. Механизм деления ядер урана.
      Протекание цепной реакции
  1. Инфографика по физике

  2. Введение

    1. Описание механического движения
    2. Прямолинейное равномерное движение
    3. Равнопеременное движение
    4. Равномерное движение по окружности
    5. Контрольная работа по теме
    1. Законы Ньютона
    2. Сила тяготения
    3. Сила упругости
    4. Силы трения
    5. Контрольная работа по теме
    1. Условия равновесия тел в ИСО
    2. Гидростатика и гидродинамика
    3. Контрольная работа по теме
    1. Импульс
    2. Механическая работа
    3. Закон сохранения энергии в механике
    4. Контрольная работа по теме
    1. Физическое описание систем с большим числом частиц
    2. Статистические закономерности МКТ
    3. Контрольная работа по теме
    1. Первый закон термодинамики
    2. Второй закон термодинамики
    3. Агрегатные состояния вещества.
      Фазовые переходы
    4. Контрольная работа по теме
    1. Электростатическое поле
    2. Работа сил электростатического поля. Разность потенциалов
    3. Электроёмкость.
      Энергия электрического поля
    1. Закон Ома для неоднородного участка цепи и замкнутой (полной) цепи
    2. Расчёт электрических цепей постоянного тока
    3. Контрольная работа по теме
    1. Электрический ток в металлах, электролитах, полупроводниках, газах и вакууме.
      Плазма
  1. Инфографика по физике

    1. Физическое описание магнитного поля
    1. Физическое описание явления электромагнитной индукции
    1. Физическое описание электромагнитных колебаний и волн
    1. Законы геометрической оптики
    1. Интерференция света
    2. Дифракция, дисперсия и поляризация света
    1. Элементы релятивистской теории
    1. Световые кванты
    2. Элементы физики атома и атомного ядра
    1. Физическое описание космических тел и систем

ГДЗ по физике 7 класс Тесты Чеботарева Решебник

Рекомендуем посмотреть

В седьмом классе начинается самый сложный период обучения в школе, ведь именно в этом году начнется курс «физика». Поэтому весьма пригодится «ГДЗ по физике за 7 класс к тестам Чеботаревой». Подросткам необходимо:

  • заучивать новый материал;
  • учить много новых и непонятных формул;
  • готовиться к контрольным и лабораторным работам.

Все это отнимает очень много времени и сил, так что ребята не могут толком отдохнуть или отвлечься на другие занятия. В итоге, многие начинают испытывать нервное напряжение, что приводит к чрезмерной утомляемости и срывам. С помощью данного сборника учащиеся получают возможность в спокойной домашней обстановке:

  1. Тщательно изучить теоретический материал.
  2. Разобрать все трудные и непонятные моменты.
  3. Правильно выполнить заданные номера.
  4. Проработать возникающие ошибки.
  5. Подготовиться к контрольным работам.

Таким образом, школьники приобретут уверенность в своих силах, станут увереннее чувствовать себя на уроках, снизят уровень стресса, который обычно возникает на проверочных испытаниях. Заодно сократится и время, затрачиваемое на выполнение д/з.

Для чего нужен ГДЗ по физике за 7 класс к тестам Чеботаревой

В сборнике представлено шесть итоговых тестов, которые соотносятся с тематическими разделами. Все номера содержат детальные решения, верные ответы и исчерпывающие пояснения, так что ученики имеют возможность:

  • разобраться в принципе реакций;
  • понять, как применяется лабораторное оборудование;
  • постигнуть алгоритм записи уравнений.

С помощью решебника подростки без труда усвоят необходимые понятия и определения, что пригодится при выполнении д/з. Школьники повысят свою успеваемость и подготовятся к любым контрольным работам. Физика является весьма интересной и познавательной наукой, которая готовит ребят ко взрослой жизни, ведь практически все явления находят отражение в нашей действительности. Однако теория достаточно сложна, поэтому школьники часто «спотыкаются» при ее изучении. А это в свою очередь ведет к проблемам при выполнении практических упражнений. С помощью «ГДЗ по физике за 7 класс к тестам Чеботаревой А. В. (Экзамен)» учащиеся освоят текущий материал в полной мере, смогут вспомнить все то, что проходили ранее, разберутся, как проходят сложные физические реакции. С решебником учеба становится намного проще.

Быстрый поиск

Темы

Введение Молекулы Взаимодействие молекул Механическое движение Скорость Инерция Масса тела Плотность вещества Расчет массы Сила Сила упругости Вес тела Единицы силы Сила тяжести на других планетах Динамометр Сила трения Давление Давление газа Передача давления жидкостями и газами Давление в жидкости и газе Расчет давления жидкости Сообщающиеся сосуды Атмосферное давление Измерение атмосферного давления Манометры Действие жидкости и газа на погруженное в них тело Архимедова сила Плавание тел Плавание судов Механическая работа Мощность Простые механизмы Момент силы Блоки Золотое правило механики Центр тяжести тела Коэффициент полезного действия механизма Энергия

Тест итоговый 1 (Варианты)

1 2 3 4

Тест итоговый 2 (Варианты)

1 2 3 4

Тест итоговый 3 (Варианты)

1 2 3 4

Тест итоговый 4 (Варианты)

1 2 3 4

Тест итоговый 5 (Варианты)

1 2 3 4

Тест итоговый 6 (Варианты)

1 2 3 4

Седьмой класс славится огромными объемами материала и большим количеством проверочных работ. Они не только проводятся после изучения каждого раздела, но и практически еженедельно учителя устраивают срез знаний. Чтобы заранее подготовиться к этим испытаниям, школьникам стоит использовать сборник готовых домашних заданий по данному предмету, где детально прописаны все аспекты материала. Благодаря данному сборнику, подростки получают возможность:

  1. Хорошо проработать непонятные моменты.
  2. Выявить и восполнить пробелы в знаниях.
  3. Запомнить все необходимые термины и определения.
  4. Улучшить качество практических работ.
  5. Повысить уровень знаний.

Систематические занятия с решебником отнимают совсем мало времени, зато приносят много пользы. С ним учащиеся быстро постигнут все премудрости столь сложной науки и будут получать исключительно хорошие оценки за все свои работы, будь они выполнены в классе или дома.

Параметры использования ГДЗ по физике за 7 класс к тестам Чеботаревой

В сборнике представлено шесть итоговых тестов по четыре варианта в каждом из них. Все задания соотносятся с материалом этого учебного года. Для того чтобы ученикам было проще преодолеть все возникающие в процессе затруднения, автор предоставил:

  • исчерпывающие пояснения;
  • детальные решения;
  • верные ответы к каждому номеру.

Используя эту информацию, школьники не только качественно подготовятся ко всем предстоящим испытаниям, но и улучшат качество д/з. Приобретя полноценные знания, ребята будут чувствовать уверенность в своих силах, что позволит им без трепета относиться к любой контрольной работе. Применяя это пособие, любой семиклассник изучит следующие темы:

  1. Физика — наука о природе. Физические явления. Физические свойства тел. Наблюдение и описание физических явлений. Физические величины. Измерения физических величин: длины, времени, температуры. Физические приборы. Международная система единиц. Точность и погрешность измерений. Физика и техника.
  2. Строение вещества. Опыты, доказывающие атомное строение вещества. Тепловое движение атомов и молекул. Броуновское движение. Диффузия в газах, жидкостях и твердых телах. Взаимодействие частиц вещества. Агрегатные состояния вещества. Модели строения твердых тел, жидкостей и газов. Объяснение свойств газов, жидкостей и твердых тел на основе молекулярно-кинетических представлений.
  3. Механическое движение. Траектория. Путь. Равномерное и неравномерное движение. Скорость. Графики зависимости пути и модуля скорости от времени движения. Инерция. Инертность тел. Взаимодействие тел. Масса тела. Измерение массы тела. Плотность вещества. Сила. Сила тяжести. Сила упругости. Закон Гука. Вес тела. Связь между силой тяжести и массой тела. Сила тяжести на других планетах. Динамометр. Сложение двух сил, направленных по одной прямой. Равнодействующая двух сил. Сила трения. Физическая природа небесных тел Солнечной системы.

Решебник по физике за 7 класс к тестам Чеботаревой А. В. (Экзамен) — помощь в учебе

Многие школьники с интересом посещают занятия по данному предмету, ведь узнают там по-настоящему интересные и актуальные вещи. Эти сведения носят не только теоретический, но и прикладной характер, что позволит ими воспользоваться в дальнейшем. Однако стоит упустить хоть одну деталь, как легко потерять связь между изучаемыми параграфами, не говоря уже о невозможности правильно выполнять многочисленные задания. Избежать всевозможных недочетов учащимся поможет сборник тестов под редакцией Чеботаревой, где приведены исчерпывающие алгоритмы по всем аспектам программы. Подростки получают возможность приобрести все необходимые знания и навыки, не затрачивая на это много времени.

Конспект по физике. Скорость | 7 класс Онлайн

Конспект по физике для 7 класса «Скорость». ВЫ УЗНАЕТЕ: Что такое скорость. Как можно определить скорость движения тел. Каковы единицы скорости. Как вычислить скорость тела при равномерном движении. Как построить графики зависимости скорости от времени. Как построить графики зависимости пути от времени. ВСПОМНИТЕ: В каком случае тело движется равномерно? Что такое путь, единицы пути?

Конспекты по физике    Учебник физики    Тесты по физике


За одно и то же время различные тела могут проходить разные расстояния. За 5 мин движения поезд прошёл большее расстояние, чем проехал велосипедист, а велосипедист большее расстояние, чем пролетела муха. Значит, одно и то же расстояние поезд проезжает быстрее, чем велосипедист, а велосипедист быстрее, чем пролетает муха.

ПОНЯТИЕ СКОРОСТИ

В физике быстроту перемещения тола характеризует такая величина, как скорость. Например, человек может за время 1 ч пройти путь 5 км, автомобиль за это же время может проехать 60 км, а самолёт пролетит 850 км. Тогда говорят, что человек движется со скоростью 5 километров в час, скорость автомобиля составляет 60 километров в час, а скорость самолёта — 850 километров в час.

В окружающем нас мире встречаются самые разные скорости. Так, черепаха может ползти со скоростью от 0,05 до 0,14 м/с. Идущий в среднем темпе человек движется со скоростью 1,4 м/с. Скорость гепарда может превышать 30 м/с. Скорость ветра во время урагана может превышать 32,6 м/с. Звук в воздухе распространяется со скоростью приблизительно 333 м/с. а Земля движется вокруг Солнца со скоростью около 30 000 м/с. Скорость света в вакууме — самая большая скорость во Вселенной равна 299 792 458 м/с.

СКОРОСТЬ ПРИ РАВНОМЕРНОМ ДВИЖЕНИИ

Скорость при равномерном движении тела показывает, какой путь проходит тело за единицу времени. Так как при равномерном движении тело за равные промежутки времени проходит равные пути, то скорость при таком движении оказывается постоянной.

Для того чтобы определить скорость тела при равномерном движении, необходимо путь, пройденный телом, разделить на время, за которое этот путь пройден.

Если обозначить величины: скорость ʋ, путь — s и время — t, то скорость рассчитывают по формуле ʋ = s / t.

ЕДИНИЦЫ СКОРОСТИ

В Международной системе единиц (СИ) за единицу скорости принимают скорость такого равномерного движения, при котором движущееся тело за 1 секунду проходит путь, равный 1 метру. Эту единицу называют 1 метр в секунду и обозначают 1 м/с.

Очень часто используются и другие единицы скорости: километр в час (км/ч), километр в секунду (км/с), сантиметр в секунду (см/с). При выборе разных единиц скорость тоже будет иметь разные численные значения.

НАПРАВЛЕНИЕ СКОРОСТИ

Величины, которые, кроме числового значения (модуля), имеют ещё и направление, называют векторными. Числовое значение векторной величины также называется модулем вектора.

Скорость, кроме числового значения, всегда имеет направление, следовательно, скорость — это векторная величина. Именно поэтому скорость обозначается как (со стрелочкой), а её модуль как ʋ (без стрелочки).

Говоря о модуле скорости, мы можем понять, насколько быстро или медленно движется физическое тело. Для решения большинства задач необходимо ещё знать, куда направлена скорость тела.

Кроме скорости, существуют и другие векторные физические величины, которые характеризуются числовым значением и направлением (с ними вы познакомитесь чуть позже). Вместе с тем существуют и другие физические величины, которые не имеют направления, а характеризуются только числовым значением. Такие физические величины называют скалярными. Примерами скалярных величин могут служить путь, объём, площадь и т. д.

ГРАФИКИ ЗАВИСИМОСТИ ПУТИ И СКОРОСТИ ОТ ВРЕМЕНИ

Путь, пройденный телом, и скорость его движения с течением времени могут изменяться. Для большей наглядности эти изменения часто изображают графически.

Для построения графиков на горизонтальной оси (абсцисс) откладывают время, а на вертикальной оси (ординат) — путь, пройденный телом, или его скорость.

График зависимости скорости от времени при равномерном движении — это прямая, параллельная оси абсцисс. Действительно, с течением времени скорость при таком движении остаётся постоянной.

График зависимости пути, пройденного телом, от времени при прямолинейном равномерном движении — это прямая, расположенная в первой четверти координатной плоскости и проведённая из начала координат. Действительно, при движении тела с постоянной скоростью с течением времени путь, пройденный телом, увеличивается. Причём эта зависимость прямая, так как она описывается уравнением s = ʋt.


Вы смотрели Конспект по физике для 7 класса «Скорость»: Что такое скорость. Как можно определить скорость движения тел. Каковы единицы скорости. Как вычислить скорость тела при равномерном движении. Как построить графики зависимости скорости от времени. Как построить графики зависимости пути от времени.

Вернуться к Списку конспектов по физике (оглавление).

Пройти онлайн-тест «Скорость, путь и время»

2.2 Скорость и скорость — физика

Раздел Цели обучения

К концу этого раздела вы сможете делать следующее:

  • Вычислять среднюю скорость объекта
  • Соотнесите перемещение и среднюю скорость

Поддержка учителей

Поддержка учителей

Цели обучения в этом разделе помогут вашим учащимся освоить следующие стандарты:

  • (4) Научные концепции. Учащийся знает и применяет законы, управляющие движением, в различных ситуациях. Ожидается, что студент:
    • (B) описывать и анализировать движение в одном измерении, используя уравнения с понятиями расстояния, смещения, скорости, средней скорости, мгновенной скорости и ускорения.

Кроме того, в Руководстве по физике для средней школы рассматривается содержание этого раздела лабораторной работы под названием «Положение и скорость объекта», а также следующие стандарты:

  • (4) Научные концепции. Учащийся знает и применяет законы, управляющие движением, в различных ситуациях. Ожидается, что студент:
    • (Б) описывать и анализировать движение в одном измерении, используя уравнения с понятиями расстояния, смещения, скорости, средней скорости, мгновенной скорости и ускорения.

Основные термины раздела

средняя скорость средняя скорость мгновенная скорость
мгновенная скорость скорость скорость

Поддержка учителей

Поддержка учителей

В этом разделе учащиеся будут применять полученные знания о расстоянии и перемещении к понятиям скорости и скорости.

[BL][OL] Прежде чем учащиеся прочитают этот раздел, попросите их привести примеры того, как они слышали употребление слова «скорость». Затем спросите их, слышали ли они слово «скорость». Объясните, что эти слова часто взаимозаменяемы в повседневной жизни, но их научные определения различны. Скажите студентам, что они узнают об этих различиях, когда будут читать этот раздел.

[AL] Объясните учащимся, что скорость, как и перемещение, является векторной величиной. Попросите их предположить, чем скорость отличается от скорости. После того, как они поделятся своими идеями, задайте вопросы, которые углубят их мыслительный процесс, например: Почему вы так думаете? Что такое пример? Как можно применить эти термины к движению, которое вы видите каждый день?

Скорость

Движение — это нечто большее, чем расстояние и смещение. Такие вопросы, как «Сколько времени занимает пеший забег?» и «Какова была скорость бегуна?» нельзя ответить без понимания других понятий. В этом разделе мы рассмотрим время, скорость и скорость, чтобы расширить наше понимание движения.

Описание того, насколько быстро или медленно движется объект, называется его скоростью. Скорость — это скорость, с которой объект меняет свое местоположение. Как и расстояние, скорость является скаляром, поскольку имеет величину, но не направление. Поскольку скорость — это скорость, она зависит от временного интервала движения. Вы можете рассчитать прошедшее время или изменение во времени ΔtΔt движения как разницу между временем окончания и временем начала

Δt=tf−t0.Δt=tf−t0.

Единицей времени в СИ является секунда (с), а единицей скорости в СИ является метр в секунду (м/с), но иногда километры в час (км/ч), мили в час (миль в час) или другие единицы измерения скорости.

Когда вы описываете скорость объекта, вы часто описываете среднее значение за определенный период времени. Средняя скорость v avg — это пройденное расстояние, деленное на время, в течение которого происходит движение.

vavg=distancetimevavg=distancetime

Вы, конечно, можете изменить уравнение, чтобы решить либо расстояние, либо время

время = distancevavg. time = distancevavg.

расстояние = срч × времярасстояние = срч × время

Предположим, например, что автомобиль проезжает 150 километров за 3,2 часа. Его средняя скорость за поездку

ср=расстояние/время=150 км3,2 ч=47 км/ч. ср=расстояние/время=150 км3,2 ч=47 км/ч.

Скорость автомобиля может увеличиваться и уменьшаться во много раз за 3,2-часовую поездку. Однако его скорость в конкретный момент времени является его мгновенной скоростью. Спидометр автомобиля описывает его мгновенную скорость.

Поддержка учителей

Поддержка учителей

[OL][AL] Предупредите учащихся, что средняя скорость не всегда является средним значением начальной и конечной скоростей объекта. Например, предположим, что автомобиль проехал расстояние 100 км. Первые 50 км он едет со скоростью 30 км/ч, а вторые 50 км – со скоростью 60 км/ч. Его средняя скорость будет равна расстоянию /(интервал времени) = (100 км)/[(50 км)/(30 км/ч) + (50 км)/(60 км/ч)] = 40 км/ч. Если бы автомобиль проехал одинаковое время 30 км и 60 км, а не равное расстояние на этих скоростях, его средняя скорость была бы 45 км/ч.

[BL][OL] Предупредите учащихся, что термины «скорость», «средняя скорость» и «мгновенная скорость» в повседневном языке часто называются просто скоростью. Подчеркните важность использования правильной терминологии в науке, чтобы избежать путаницы и правильно передать идеи.

Рисунок 2,8 За 30-минутную поездку туда и обратно до магазина общее пройденное расстояние составляет 6 км. Средняя скорость 12 км/ч. Перемещение для кругового рейса равно нулю, потому что нет чистого изменения положения.

Рабочий пример

Вычисление средней скорости

Мрамор катится на 5,2 м за 1,8 с. Какова была средняя скорость шарика?

Стратегия

Мы знаем расстояние, которое проходит шарик, 5,2 м, и интервал времени, 1,8 с. Мы можем использовать эти значения в уравнении средней скорости.

Решение

vavg=distancetime=5,2 м1,8 с=2,9 м/svavg=distancetime=5,2 м1,8 с=2,9 м/с отвечать. Мы можем проверить разумность ответа, оценив: 5 метров разделить на 2 секунды равно 2,5 м/с. Поскольку 2,5 м/с близко к 2,9м/с, ответ разумный. Речь идет о скорости быстрой ходьбы, так что это тоже имеет смысл.

Практические задачи

9.

Питчер бросает бейсбольный мяч с насыпи питчера на домашнюю тарелку за 0,46 с. Расстояние 18,4 м. Какова была средняя скорость бейсбольного мяча?

  1. 40 м/с
  2. — 40 м/с
  3. 0,03 м/с
  4. 8,5 м/с

10.

Кэсси шла к дому своей подруги со средней скоростью 1,40 м/с. Расстояние между домами 205 м. Сколько времени заняла у нее поездка?

  1. 146 с

  2. 0,01 с

  3. 2,50 мин

  4. 287 с

Скорость

Векторная версия скорости — это скорость. Скорость описывает скорость и направление объекта. Как и в случае со скоростью, полезно описывать либо среднюю скорость за период времени, либо скорость в конкретный момент. Средняя скорость равна смещению, деленному на время, в течение которого происходит смещение.

vavg=displacementtime=ΔdΔt=df−d0tf−t0vavg=displacementtime=ΔdΔt=df−d0tf−t0

Скорость, как и скорость, измеряется в единицах СИ в метрах в секунду (м/с), но поскольку это вектор, вы также должен включать направление. Кроме того, переменная v для скорости выделена жирным шрифтом, потому что это вектор, в отличие от переменной v для скорости, которая выделена курсивом, потому что это скалярная величина.

Советы для успеха

Важно помнить, что средняя скорость — это не то же самое, что средняя скорость без направления. Как мы видели со смещением и расстоянием в предыдущем разделе, изменения направления во временном интервале оказывают большее влияние на скорость и скорость.

Предположим, что пассажир двигается к задней части самолета со средней скоростью –4 м/с. Мы не можем сказать по средней скорости, остановился ли пассажир на мгновение или дал задний ход, прежде чем он добрался до задней части самолета. Чтобы получить больше деталей, мы должны рассмотреть меньшие сегменты пути за более короткие интервалы времени, такие как те, что показаны на рис. 2.9. Если вы рассматриваете бесконечно малые интервалы, вы можете определить мгновенную скорость, то есть скорость в определенный момент времени. Мгновенная скорость и средняя скорость одинаковы, если скорость постоянна.

Рисунок 2,9 На диаграмме показана более подробная запись пассажира самолета, направляющегося к задней части самолета, с указанием меньших сегментов его поездки.

Ранее вы читали, что пройденное расстояние может отличаться от величины перемещения. Точно так же скорость может отличаться от величины скорости. Например, вы едете в магазин и через полчаса возвращаетесь домой. Если одометр вашего автомобиля показывает, что общее пройденное расстояние составило 6 км, значит, ваша средняя скорость составила 12 км/ч. Однако ваша средняя скорость была равна нулю, потому что ваше смещение за кругосветное путешествие равно нулю.

Смотреть физику

Вычисление средней скорости или скорости

В этом видео рассматриваются векторы и скаляры, а также описывается, как рассчитать среднюю скорость и среднюю скорость, когда известны перемещение и изменение во времени. В видео также рассматривается, как преобразовать км/ч в м/с.

Что из следующего полностью описывает вектор и скалярную величину и правильно дает пример каждого из них?

  1. Скалярная величина полностью описывается своей величиной, тогда как для полного описания вектору необходимы и величина, и направление. Перемещение является примером скалярной величины, а время — примером векторной величины.

  2. Скалярная величина полностью описывается своей величиной, тогда как для полного описания вектору необходимы и величина, и направление. Время — пример скалярной величины, а перемещение — пример векторной величины.

  3. Скалярная величина полностью описывается своей величиной и направлением, в то время как вектору для полного описания требуется только величина. Перемещение является примером скалярной величины, а время — примером векторной величины.

  4. Скалярная величина полностью описывается своей величиной и направлением, в то время как вектору для полного описания требуется только величина. Время — пример скалярной величины, а перемещение — пример векторной величины.

Поддержка учителей

Поддержка учителей

В этом видео хорошо видна разница между векторами и скалярами. Студент знакомится с идеей использования «s» для обозначения перемещения, которое вы можете поощрять или не поощрять. Прежде чем учащиеся посмотрят видео, обратите внимание на то, что преподаватель использует s→s→ для смещения вместо d, как в этом тексте. Объясните, что использование маленьких стрелок над переменными является распространенным способом обозначения векторов в курсах физики более высокого уровня. Предупредите учащихся, что в этом видео не используются общепринятые сокращения для часов и секунд. Напомните учащимся, что в своей работе они должны использовать сокращения h для обозначения часов и s для обозначения секунд.

Рабочий пример

Вычисление средней скорости

Студент переместился на 304 м на север за 180 с. Какова была средняя скорость студента?

Стратегия

Мы знаем, что смещение 304 м на север и время 180 с. Для решения задачи можно использовать формулу средней скорости.

Решение

ср=ΔdΔt=304 м180 с=1,7 м/с север vср=ΔdΔt=304 м180 с=1,7 м/с север

2,1

Обсуждение

Поскольку средняя скорость является векторной величиной, вы должны указать в ответе не только величину, но и направление. Обратите внимание, однако, что направление можно опустить до конца, чтобы не загромождать задачу. Обратите внимание на значащие цифры в задаче. Расстояние 304 м имеет три значащих цифры, а временной интервал 180 с — только две, поэтому частное должно иметь только две значащие цифры.

Советы для успеха

Обратите внимание на способ представления скаляров и векторов. В этой книге d представляет собой расстояние и перемещение. Точно так же v представляет скорость, а v представляет скорость. Переменная, не выделенная жирным шрифтом, указывает на скалярную величину, а переменная, выделенная жирным шрифтом, указывает на векторную величину. Векторы иногда представляются маленькими стрелками над переменной.

Поддержка учителей

Поддержка учителей

Используйте это задание, чтобы подчеркнуть важность использования правильного количества значащих цифр в вычислениях. Некоторые учащиеся склонны включать много цифр в свои окончательные расчеты. Они ошибочно полагают, что повышают точность своего ответа, записывая многие из цифр, показанных на калькуляторе. Обратите внимание, что это приводит к ошибкам в расчетах. В более сложных расчетах эти ошибки могут распространяться и приводить к неправильному окончательному ответу. Вместо этого напомните учащимся всегда использовать одну или две дополнительные цифры в промежуточных вычислениях и округлять окончательный ответ до правильного количества значащих цифр.

Рабочий пример

Решение для смещения, когда известны средняя скорость и время

Лейла бежит трусцой со средней скоростью 2,4 м/с на восток. Каково ее водоизмещение через 46 секунд?

Стратегия

Мы знаем, что средняя скорость Лейлы составляет 2,4 м/с на восток, а временной интервал равен 46 секундам. Мы можем изменить формулу средней скорости, чтобы найти смещение.

Решение

vср=ΔdΔtΔd=vaсрΔt=(2,4 м/с)(46 с)=1,1×102 м истср=ΔdΔtΔd=vaсрΔt=(2,4 м/с)(46 с)=1,1×102 м восток

2.2

Обсуждение

Ответ примерно 110 м на восток, что является разумным смещением для чуть менее минуты бега трусцой. Калькулятор показывает ответ как 110,4 м. Мы решили написать ответ, используя экспоненциальную запись, потому что хотели, чтобы было ясно, что мы использовали только две значащие цифры.

Советы для успеха

Анализ размерностей — хороший способ определить, правильно ли вы решили задачу. Запишите расчет, используя только единицы измерения, чтобы убедиться, что они совпадают по разные стороны от знака равенства. В рабочем примере у вас есть
м = (м/с)(с). Поскольку секунды находятся в знаменателе средней скорости и в числителе времени, единица измерения сокращается, оставляя только m и, конечно же, m = m.

Рабочий пример

Решение для времени, когда известны перемещение и средняя скорость

Филипп идет по прямой дорожке от своего дома до школы. Сколько времени потребуется ему, чтобы добраться до школы, если он пройдет 428 м на запад со средней скоростью 1,7 м/с на запад?

Стратегия

Мы знаем, что перемещение Филиппа составляет 428 м к западу, а его средняя скорость — 1,7 м/с к западу. Мы можем рассчитать время, необходимое для поездки, изменив уравнение средней скорости.

Solution

vavg=ΔdΔtΔt=Δdvavg=428 m1.7 m/s=2.5×102 svavg=ΔdΔtΔt=Δdvavg=428 m1.7 m/s=2.5×102 s

2.3

Discussion

Here again we пришлось использовать научную запись, потому что ответ мог иметь только две значащие цифры. Поскольку время является скаляром, ответ включает только величину, а не направление.

Практические задачи

11.

Дальнобойщик едет по прямому шоссе 0,25 ч со смещением 16 км на юг. Какова средняя скорость дальнобойщика?

  1. 4 км/ч на север

  2. 4 км/ч на юг

  3. 64 км/ч север

  4. 64 км/ч на юг

12.

Птица перелетает со средней скоростью 7,5 м/с на восток с одной ветки на другую за 2,4 с. Затем он делает паузу перед полетом со средней скоростью 6,8 м/с на восток в течение 3,5 с к другой ветке. Каково полное перемещение птицы от исходной точки?

  1. 42 м на запад
  2. 6 м на запад
  3. 6 м на восток
  4. 42 м на восток

Виртуальная физика

Ходячий человек

В этой симуляции вы будете наводить курсор на человека и перемещать его сначала в одном направлении, а затем в противоположном. Держите вкладку Introduction активной. Вы можете использовать вкладку Charts после того, как узнаете о построении графиков движения позже в этой главе. Внимательно следите за знаком чисел в полях положения и скорости. На данный момент игнорируйте поле ускорения. Посмотрите, сможете ли вы сделать положение человека положительным, а скорость отрицательной. Затем посмотрите, можете ли вы сделать наоборот.

Проверка захвата

Какая ситуация правильно описывает ситуацию, когда положение движущегося человека было отрицательным, но его скорость была положительной?

  1. Человек движется к 0 слева от 0
  2. Человек движется к 0 справа от 0
  3. Человек уходит от 0 слева от 0
  4. Человек уходит от 0 справа от 0

Поддержка учителей

Поддержка учителей

Это мощная интерактивная анимация, которую можно использовать во многих уроках. На этом этапе его можно использовать, чтобы показать, что смещение может быть как положительным, так и отрицательным. Это также может показать, что когда смещение отрицательно, скорость может быть как положительной, так и отрицательной. Позже его можно использовать, чтобы показать, что скорость и ускорение могут иметь разные знаки. Настоятельно рекомендуется, чтобы вы удерживали учащихся на уровне 9. 0081 Введение табл. Вкладку Charts можно использовать после того, как учащиеся узнают о построении графиков движения позже в этой главе.

Проверьте свое понимание

13.

Два бегуна, движущиеся по одной и той же прямой дорожке, начинают и заканчивают свой бег в одно и то же время. На полпути они имеют разные мгновенные скорости. Могут ли их средние скорости за весь путь быть одинаковыми?

  1. Да, потому что средняя скорость зависит от чистого или полного водоизмещения.

  2. Да, потому что средняя скорость зависит от общего пройденного пути.

  3. Нет, потому что скорости обоих бегунов должны оставаться одинаковыми на протяжении всего пути.

  4. Нет, потому что мгновенные скорости бегунов должны оставаться одинаковыми в средней точке, но могут различаться в других точках.

14.

Если вы делите общее расстояние, пройденное за автомобильную поездку (определяемое одометром), на время в пути, вычисляете ли вы среднюю скорость или величину средней скорости, и при каких обстоятельствах эти две величины такой же?

  1. Средняя скорость. Оба одинаковы, когда автомобиль движется с постоянной скоростью и меняет направление.
  2. Средняя скорость. Оба одинаковы, когда скорость постоянна и автомобиль не меняет своего направления.
  3. Величина средней скорости. Оба одинаковы, когда автомобиль движется с постоянной скоростью.
  4. Величина средней скорости. Оба одинаковы, когда автомобиль не меняет своего направления.

15.

Может ли средняя скорость быть отрицательной?

  1. Да, если чистый водоизмещение отрицательное.

  2. Да, если направление объекта меняется во время движения.

  3. Нет, потому что средняя скорость описывает только величину, а не направление движения.

  4. Нет, потому что средняя скорость описывает величину только в положительном направлении движения.

Поддержка учителей

Поддержка учителей

Используйте Проверьте свое понимание вопроса для оценки достижений учащихся по разделам целей обучения. Если учащиеся испытывают трудности с выполнением определенной задачи, тест «Проверка вашего понимания» поможет определить, что именно, и направит учащихся к соответствующему содержанию. Элементы оценки в TUTOR позволят вам переоценить.

Скорость и скорость

Скорость и Скорость

Скорость — это то, насколько быстро что-то движется.

Скорость — это скорость в направлении .

 

Если сказать, что собака Ариэль бежит со скоростью 9 км/ч, то (километров в час) — это скорость.

Но если сказать, что он бежит 9 км/ч на запад , то это скорость.

 

  Скорость Скорость
Имеет: величина величина и направление
Пример: 60 км/ч 60 км/ч Север
Пример: 5 м/с 5 м/с вверх

Представьте себе что-то очень быстро движущееся вперед и назад: оно имеет высокую скорость, но низкую (или нулевую) скорость.

Скорость

Скорость измеряется как расстояние, пройденное за время.

Скорость = Расстояние Время

Пример: Автомобиль проезжает 50 км за один час.

Средняя скорость 50 км/ч (50 км/ч)

Скорость = Расстояние Время знак равно 50 км 1 час

Мы также можем использовать эти символы:

Скорость = Δs Δt

Где Δ (« Дельта ») означает «изменение», а

  • s означает расстояние («s» вместо «пробел»)
  • t означает время

Пример: Вы пробежали 360 м за 60 секунд.

Скорость = Δs Δt

= 360 м 60 секунд

= 6 м 1 секунда

Итак, ваша скорость равна 6 метрам в секунду (6 м/с).

Единицы

Скорость обычно измеряется в:

  • метров в секунду (м/с или м с -1 ), или
  • километров в час (км/ч или км ч -1 )

Км равен 1000 м, а в часе 3600 секунд, поэтому мы можем преобразовать следующим образом (см. Метод преобразования единиц, чтобы узнать больше):

1 м 1 с × 1 км 1000 м × 3600 с 1 час знак равно 3600 м·км·с 1000 с · м · ч знак равно 3,6 км 1 час

Итак, 1 м/с равен 3,6 км/ч

Пример: сколько будет 20 м/с в км/ч?

20 м/с × 3,6 км/ч 1 м/с = 72 км/ч

Пример: Сколько будет 120 км/ч в м/с?

120 км/ч × 1 м/с 3,6 км/ч = 33,333… м/с

Средняя и мгновенная скорость

Примеры до сих пор вычисляют среднюю скорость : расстояние, которое что-то проходит за определенный период времени.

Но со временем скорость может измениться. Автомобиль может двигаться быстрее и медленнее, может даже останавливаться на светофоре.

Итак, есть также мгновенная скорость : скорость в момент во времени. Мы можем попытаться измерить его, используя очень короткий промежуток времени (чем короче, тем лучше).

Пример. Сэм использует секундомер и измеряет 1,6 секунды, когда автомобиль проезжает между двумя столбами, расстояние между которыми составляет 20 м. Какова мгновенная скорость

?

Ну, мы точно не знаем, так как машина могла ускоряться или замедляться в это время, но мы можем оценить:

20 м 1,6 с = 12,5 м/с = 45 км/ч

Это действительно все еще средняя, ​​но близкая к мгновенной скорость.

Постоянная скорость

Когда скорость не меняется, она постоянна .

Для постоянной скорости средняя и мгновенная скорости совпадают.

Скорость

Скорость — это скорость в направлении .

На самом деле это вектор …

… так как он имеет величину и направление

Поскольку направление важно, скорость использует смещение вместо расстояния:

Скорость = Расстояние Время

Скорость = Рабочий объем Время в направлении.

Пример. Вы идете от дома до магазина за 100 секунд. Какова ваша скорость и какова ваша скорость?

Скорость = 220 м 100 с = 2,2 м/с

Скорость = 130 м 100 с Восток = 1,3 м/с Восток

Вы забыли свои деньги, поэтому разворачиваетесь и возвращаетесь домой еще через 120 секунд: какова ваша скорость туда и обратно?

Общее время 100 с + 120 с = 220 с:

Скорость = 440 м 220 с = 2,0 м/с

Скорость = 0 м 220 с = 0 м/с

Да, скорость равна нулю, так как вы оказались там, где начали.

Узнайте больше на сайте Vectors.

Родственник

Движение относительное. Когда мы говорим, что что-то «покоится» или «движется со скоростью 4 м/с», мы забываем сказать «относительно меня» или «относительно земли» и т. д.

Подумайте об этом: вы действительно стоите на месте? Вы находитесь на планете Земля, которая вращается со скоростью 40 075 км в день (около 1675 км/ч или 465 м/с) и движется вокруг Солнца со скоростью около 100 000 км/ч, которая сама движется через Галактику.

В следующий раз, когда будете гулять, представьте, что вы неподвижны, а мир движется у вас под ногами. Прекрасно себя чувствует.

Все относительно!

 

 

Скорость и скорость — Гиперучебник по физике

[закрыть]

скорость

В чем разница между двумя одинаковыми объектами, движущимися с разной скоростью? Почти все знают, что тот, кто движется быстрее (тот, у кого больше скорость), пройдет дальше, чем тот, кто движется медленнее, за то же время. Либо так, либо вам скажут, что тот, кто движется быстрее, доберется до места раньше, чем тот, кто медленнее. Какой бы ни была скорость, она включает в себя и расстояние, и время. «Быстрее» означает «дальше» (большее расстояние) или «скорее» (меньше времени).

Удвоение скорости означает удвоение пройденного расстояния за заданный промежуток времени. Удвоение скорости также означало бы сокращение вдвое времени, необходимого для преодоления заданного расстояния. Если вы немного разбираетесь в математике, эти утверждения будут значимыми и полезными. (Символ v используется для обозначения скорости из-за связи между скоростью и скоростью, которая будет вскоре обсуждаться.)

  • Скорость прямо пропорциональна расстоянию, когда время постоянно: v с ( t константа)
  • Скорость обратно пропорциональна времени при постоянном расстоянии: v 1 t ( с  константа)

Объединение этих двух правил дает определение скорости в символической форме.

v  =  с
т
☞ Это не окончательное определение.

Не нравятся символы? Тогда вот еще один способ определить скорость. Скорость скорость изменения расстояния со временем.

Чтобы вычислить скорость объекта, мы должны знать, как далеко он ушел и сколько времени потребовалось, чтобы туда добраться. «Дальше» и «ранее» соответствуют «быстрее». Допустим, вы поехали на машине из Нью-Йорка в Бостон. Расстояние по дороге составляет примерно 300 км (200 миль). Если поездка занимает четыре часа, какова была ваша скорость? Применение приведенной выше формулы дает…

v  =  с  ≈  300 км = 75 км/ч
т 4 часа

Это ответ, который дает нам уравнение, но насколько он верен? Было ли 75 км/ч 90 514 скоростью автомобиля 90 082? Да, конечно, это было… Ну, может быть, я предполагаю… Нет, это не могла быть скорость . Если только вы не живете в мире, где автомобили имеют какой-то исключительный круиз-контроль и транспортные потоки регулируются каким-то идеальным образом, ваша скорость во время этого гипотетического путешествия, безусловно, должна была варьироваться. Таким образом, число, рассчитанное выше, не равно 9. 0514 скорость автомобиля, это средняя скорость за весь путь. Чтобы подчеркнуть этот момент, уравнение иногда модифицируют следующим образом…

v  =  с
т

Черта между и указывает среднее или среднее значение, а символ ∆ (дельта) указывает на изменение. Прочитайте это как «Vee bar — это дельта-эсс над дельта-тройником». Это количество мы рассчитали для нашей гипотетической поездки.

Напротив, спидометр автомобиля показывает его мгновенную скорость , то есть скорость, определяемую за очень небольшой интервал времени — мгновение. В идеале этот интервал должен быть как можно ближе к нулю, но на самом деле мы ограничены чувствительностью наших измерительных приборов. Мысленно, однако, можно представить вычисление средней скорости за все меньшие интервалы времени, пока мы не вычислим мгновенную скорость. Эта идея записывается символически как…

v  = 
лим
t →0
с  =  дс
т дт

или на языке исчисления скорость есть первая производная расстояния по времени.

Если вы не занимались исчислением, не парьтесь слишком сильно с этим определением. Есть и другие, более простые способы определения мгновенной скорости движущегося объекта. На графике расстояние-время скорость соответствует наклону, и, таким образом, мгновенная скорость объекта с непостоянной скоростью может быть найдена по наклону линии, касательной к его кривой. Мы займемся этим позже в этой книге.

скорость

Чтобы вычислить скорость объекта, нам нужно знать, как далеко он ушел и сколько времени потребовалось, чтобы добраться туда. Тогда мудрый человек спросил бы…

Что вы подразумеваете под как далеко ? Вам нужно расстояние или смещение ?

Мудрый человек, когда-то давно

Ваш выбор ответа на этот вопрос определяет, что вы рассчитываете — скорость или скорость.

  • Средняя скорость — скорость изменения расстояния со временем.
  • Средняя скорость — скорость изменения смещения со временем.

А для людей, занимающихся расчетами…

  • Мгновенная скорость является первой производной расстояния по времени.
  • Мгновенная скорость является первой производной перемещения по времени.

Скорость и скорость связаны почти так же, как связаны расстояние и перемещение. Скорость — это скаляр, а скорость — это вектор. Скорость получает символ v (курсив), а скорость получает символ v (жирный шрифт). Средние значения получают полосу над символом.

средняя
скорость
 
v  =  с
т
мгновенная
скорость
 
v  = 
лим
t →0
с  =  дс
т дт
среднее
скорость
 
v  =  с
т
мгновенная
скорость
 
v  = 
лим
t →0
с  =  д с
т дт

Смещение измеряется по кратчайшему пути между двумя точками, и его величина всегда меньше или равна расстоянию. Величина смещения приближается к расстоянию, когда расстояние приближается к нулю. То есть расстояние и смещение фактически одинаковы (имеют одинаковую величину), когда исследуемый интервал «небольшой». Поскольку скорость основана на расстоянии, а скорость основана на смещении, эти две величины фактически одинаковы (имеют одинаковую величину), когда временной интервал «небольшой» или, говоря языком исчисления, величина средней скорости объекта приближается. его средняя скорость по мере приближения интервала времени к нулю.

t  → 0 ⇒  против  → | против |

Мгновенная скорость объекта равна величине его мгновенной скорости.

v  = | против |

Скорость говорит вам, насколько быстро. Скорость говорит вам, как быстро и в каком направлении.

шт.

Скорость и скорость измеряются в одних и тех же единицах измерения. Единицей расстояния и перемещения в СИ является метр. Единицей времени в СИ является секунда. Единицей скорости и скорости в СИ является отношение двух — 9.0820 метров в секунду .



м  =  м

с с

Этот прибор редко используется за пределами научных и академических кругов. Большинство людей на этой планете измеряют скорость в километров в час (км/ч или км/ч). Соединенные Штаты являются исключением в том, что мы используем более старую милю в час (миль/ч или миль в час). Давайте определим коэффициенты преобразования, чтобы мы могли связать скорость, измеренную в м/с, с более привычными единицами измерения.

1 9079 миль/час0043
1 км/ч = 1 км   1000 м   1 час  
1 час 1 км 3600 с  
1 км/ч = 0,2777… м/с ≈ ¼ м/с  
 
1 миля   1609 м   1 час  
1 час 1 миля 3600 с  
1 час в час = 0,4469… м/с ≈ ½ м/с  
 

Десятичные значения, показанные выше, точны до четырех значащих цифр, но дробные значения следует рассматривать только как эмпирические правила (1 км/ч на самом деле больше похож на 2 7 м/с, чем 1 4 м/с и 1 миль в час больше похоже на 4 9 м/с, чем на 1 29086 29008).

Отношение любой единицы расстояния к любой единице времени является единицей скорости.

  • Скорость кораблей, самолетов и ракет часто указывается в узлах . Один узел равен одной морской миле в час — морская миля составляет 1852 м или 6076 футов, а час — 3600 с. НАСА до сих пор сообщает скорость своих ракет в узлах и расстояние до них в морских милях. Один узел составляет примерно 0,5144 м/с.
  • Самые низкие скорости измеряются в течение самых длинных периодов времени. Континентальные плиты ползут по поверхности Земли с геологически медленной скоростью 1–10 см/год или 1–10 м/столетие — примерно с такой же скоростью растут ногти и волосы.
  • Аудиокассета движется со скоростью 1⅞ дюймов в секунду (ips). Когда магнитная лента была впервые изобретена, ее наматывали на открытые катушки, как кинопленку. Эти ранние катушечные магнитофоны воспроизводили ленту со скоростью 15 кадров в секунду. Более поздние модели также могли записывать с половиной этой скорости (7½ дюймов в секунду), затем с половиной этой скорости (3¾ дюймов в секунду), а затем с половиной этой скорости (1⅞ дюймов в секунду). Когда формулировался стандарт аудиокассет, было решено, что последнего из этих значений будет достаточно для нового носителя. Один дюйм в секунду по определению равен 0,0254 м/с.

Иногда скорость объекта описывается относительно скорости чего-то другого; желательно какое-то физическое явление.

  • Аэродинамика изучает движущийся воздух и то, как объекты взаимодействуют с ним. В этой области скорость объекта часто измеряется относительно скорости звука . Это отношение известно как число Маха . Скорость звука составляет примерно 295 м/с (660 миль/ч) на высоте, на которой обычно летают коммерческие реактивные самолеты. Выведенный из эксплуатации сверхзвуковой Concorde British Airways и Air France двигался со скоростью 600 м/с (1340 миль в час). Простое деление показывает, что эта скорость примерно вдвое превышает скорость звука или 2,0 Маха, что является исключительно быстрым. Для сравнения, Боинг 777 развивает крейсерскую скорость 248 м/с (555 миль в час) или 0,8 Маха, что только кажется медленным по сравнению с Конкордом.
  • скорость света в вакууме определяется в системе СИ как 299 792 458 м/с (около миллиарда км/ч). Обычно это указывается с более разумной точностью как 3,00 × 10 8 м/с. Скорость света в вакууме обозначается символом c (курсивом) при использовании в уравнении и c (римским шрифтом) при использовании в качестве единицы измерения. Скорость света в вакууме является универсальным пределом, поэтому реальные объекты всегда движутся медленнее, чем c . Он часто используется в физике элементарных частиц и астрономии далеких объектов. Самые далекие наблюдаемые объекты — квазары; сокращение от «квазизвездные радиообъекты». Визуально они похожи на звезды (приставка «квази» означает «похожие»), но излучают гораздо больше энергии, чем любая звезда. Они лежат на окраинах наблюдаемой Вселенной и с невероятной скоростью уносятся от нас. Самые далекие квазары удаляются от нас почти на 0,9в. Кстати, символ c был выбран не потому, что скорость света является универсальной константой (которой она и является), а потому, что это первая буква латинского слова, обозначающего быстроту — celeritas .
.
Выбранные скорости (от самой низкой до самой высокой)
м/с км/ч устройство, событие, явление, процесс
10 −9 ~10 −8 континентальные пластины, рост волос, рост ногтей
10 −4 человеческие сперматозоиды
10 −3 улитки
0,013 0,045 розлив кетчупа из бутылки
10 −1 ленивцы, черепахи, черепахи
0,65–1,29 2,34–4,64 тараканы
1 3,6 нервные импульсы, немиелинизированные клетки
1 3,6 океанские течения
0,06–1,14 0,22–4,10 ламантины
1,3 4,8 человек, типичный темп ходьбы
2,391 8. 608 самый быстрый человек: плавание (Сезар Сьело)
8 30 максимальная комфортная скорость лифта
10 40 дельфины, морские свинки, киты
10 40 падающие капли дождя
10.422 37.520 самый быстрый человек: бег (Усэйн Болт)
12 43 волна стадиона
12 44 пробка для шампанского
15.223 54.803 самый быстрый человек: катание на коньках (Павел Кулижников)
20 70 кролики, зайцы, лошади, борзые, тунец, акулы
30 100 типичное ограничение скорости на автостраде
33 118 гепарды
34,42 123,9 самый быстрый человек: поле для софтбола (Моника Эбботт)
40 140 падающий град
42,47 152,9 самый быстрый человек: метание летающего диска (Саймон Лизотт)
46,98 169,1 самый быстрый человек: бейсбольное поле (Арольдис Чепмен)
55 200 предельная скорость типичного парашютиста
70,8217 254,958 самый быстрый человек: катание на лыжах (Иван Оригон)
73,06 263 самый быстрый человек: подача в теннисе (Сэм Грот)
80 290 сапсан в пикировании
82 295 очень быстрый мяч для гольфа
82. 211 296,00 самый быстрый человек: велосипедист (Дениз Коренек Мюллер)
33–83 120–300 ураган, максимальная устойчивая скорость ветра
30–90 105–330 торнадо, максимальная устойчивая скорость ветра
100 360 нервные импульсы, миелинизированные клетки
113,2 407,5 максимальный порыв приземного ветра (остров Барроу, Австралия)
118,3 426 самый быстрый человек: победа в бадминтоне (Мадс Пилер Колдинг)
124,22 447,19 самый быстрый дорожный автомобиль (Koenigsegg Agera RS)
142,89 511,11 самый быстрый корабль (Дух Австралии)
159,7 574,8 самый быстрый поезд (Train à Grande Vitesse)
168,249 605,697 самый быстрый мотоцикл (Top 1 Ack Attack)
200 700 цунами
250 900 коммерческий реактивный самолет
331 1 190 скорость звука в воздухе, СТП
340 1 225 скорость звука в воздухе на уровне моря
341. 4031 1 229,051 самый быстрый экспериментальный автомобиль (Thrust SSC)
343 1 235 скорость звука в воздухе при комнатной температуре
377,1 1 357,6 самый быстрый человек: прыжки с парашютом (Феликс Баумгартнер)
980.433 3 529,56 самый быстрый самолет (SR-71 Blackbird)
180–1 200 650–4 400 пули
1 500 5 400 скорость звука в воде
2000 6000 сейсмические волны
6 900 25 000 скорость детонации тротила
8000 29 000 космический шаттл на орбите
11 094 39 938 самый быстрый пилотируемый космический корабль (Аполлон 10)
11 180 40 250 космическая скорость на поверхности Земли
13 790 49 600 Космический зонд New Horizons
15 400 55 400 Космический зонд «Вояджер-2»
17 000 61 200 Космический зонд «Вояджер 1»
29 790 107 200 Земля на орбите
201 000 724 000 самый быстрый беспилотный космический корабль (Parker Solar Probe)
248 000 892 000 Солнце движется по Млечному Пути
300 000 1 100 000 солнечный ветер у земли
370 000 1 330 000 Млечный Путь через космический микроволновый фон
60 000 000 216 000 000 Project Starshot, предлагаемый межзвездный космический зонд
124 000 000 446 000 000 скорость света в алмазе
225 000 000 810 000 000 скорость света в воде
299 792 369 1 079 252 530 протонов и антипротонов в Тэватроне, Фермилаб
299 792 455 1 079 252 840 протона в Большом адронном коллайдере, ЦЕРН
299 792 458 1 079 252 850 скорость света в вакууме

Вопросы критического мышления по физике — AP Central

Введение

Акцент на концептуальном понимании, методах решения задач и лабораторных работах в курсах AP Physics требует использования различных аудиовизуальных средств и демонстраций для ясного и глубокого понимания обсуждаемых тем. Постановка сложных вопросов по физике будет стимулировать навыки критического мышления у учащихся. Следующие действия помогут учащимся лучше понять концепции курсов и положительно повлияют на их результаты на экзамене. Эти вопросы сосредоточены на следующих темах критического мышления:

Часть 1

Единицы и размеры: 1-3
Расстояние и перемещение: 4-6
Кинематика и движение снаряда: 7-13
Круговое движение: 14
Вес и невесомость: 15-18
Угловой момент: 19
Электричество и магнетизм: 20- 21

Часть 2

Теплофизика: 22–24
Гидромеханика: 25–32
Волны и оптика: 33–35
Современная физика: 36

Вопросы критического мышления по физике: Часть 1

  1. 1 доллар США = 100 центов
    = 10 центов x 10 центов
    = $(1/10) x $(1/10)
    = $(1/100)
    = 1 цент

    Ответ: Неверное использование единиц измерения. На втором шаге эффективная единица ¢ 2 не совпадает с $ в левой части. Опять же, единица 2 долларов на шаге 3 изменяется на доллары на четвертом шаге.
     

  2. Эйфелева башня имеет массу 10 000 000 кг. Модель башни в масштабе 100:1, изготовленная из того же материала, будет иметь массу

    1. 100 000 кг
    2. 10 000 кг
    3. 1000 кг
    4. 100 кг
    5. 10 кг
    6. 1 кг

    Ответ: (E) 10 кг

    Некоторые учащиеся могут перейти к ответу 100 000 кг, думая, что модель будет весить 1/100 реальной башни. Однако, если высота модели составляет 1/100 высоты башни, все ее размеры равны 1/100. Следовательно, модель равна (1/100) x (1/100) x (1/100) = 1 миллионная часть объема фактической башни (независимо от того, какой формы башня). Так что если модель сделана из того же материала, что и башня, то ее масса будет равна 1 миллионной массы башни, т. е. 10 кг.
     

  3. Трое мужчин — A, B и C — пересеклись в лесу холодной ночью. Они решили разжечь костер, чтобы отдохнуть, и отправились собирать дрова. A вернулся с 5 бревнами, B принес 3 бревна, а C вернулся ни с чем. С попросил дать ему отдохнуть у костра и пообещал заплатить им немного денег утром. Утром С заплатил им 8 долларов. Как А и В должны справедливо разделить деньги?

    1. A 7 долларов; В $1
    2. 6 долларов США; Б $2
    3. 5 долларов США; В $3
    4. 4 доллара США; В $4
    5. Ни один из этих

    Ответ: (A) A $7; Б $1

    Все трое в равной степени получают пользу от огня из 8 поленьев. Каждый мужчина использовал 8/3 бревна за ночь. Следовательно,
    А дал 5 — 8/3 = 7/3 бревен древесины.
    B внес 3 — 8/3 = 1/3 бревна дерева.
    Следовательно, они должны разделить 8 долларов в пропорции 7/3:1/3 или 7:1.
     

  4. Насекомое карабкается по вертикальной стене высотой 30 футов. Начиная снизу, днем ​​он поднимается на 3 фута, а ночью соскальзывает на 2 фута. Через сколько дней он достигнет вершины стены?

    1. 31 день
    2. 30 дней
    3. 29 дней
    4. 28 дней
    5. 27 дней
    6. Никогда

    Ответ: (D) 28 дней

    Некоторые учащиеся могут ответить 30 дней, утверждая, что насекомое вырастает на 1 фут в день. Но за 27 дней он поднимется на 27 футов, а на 28-й день преодолеет оставшиеся 3 фута, чтобы достичь вершины.
     

  5. Человек где-то на земле проходит 10 миль. на юг, затем 10 миль. на восток, затем 10 миль. к северу. Он вернулся в исходную точку. В каком месте на земле он?

    Ответ: На Северном полюсе есть одно решение, а на Южном полюсе — бесконечное количество решений, как показано на диаграммах ниже.

  6. Путешественник начал подниматься в гору в 6:00 утра и либо продолжал подниматься, либо отдыхал в каком-то месте (местах). Он достиг вершины в 18:00. Там он отдыхал следующие 12 часов. На следующий день в 6 часов утра он начал движение по тому же пути. Он либо двигался вниз, либо отдыхал в каком-то месте. Что касается поездок вверх и вниз, сколько раз он был в одном и том же месте в одно и то же время?

    1. Никогда
    2. Хотя бы один раз
    3. Один и только один раз
    4. Не более одного раза
    5. Только дважды
    6. Ни один из этих

    Ответ: (C) Один и только один раз

    Метод 1. Нарисуйте график зависимости x от t для туриста, где t варьируется от 6:00 до 18:00. на два дня. Графики для двух поездок будут пересекаться только для одного значения x.

    Метод 2. Представьте, что когда турист начинает подъем, есть «виртуальный турист», который начинает спуск в 6:00 утра. Легко видеть, что два «туриста» встретятся один и только один раз.
     

  7. Мистер Физ возвращается домой со скоростью 2 мили в час со своей собакой Икс. Он выпускает Икса, когда они все еще в 3 милях от его дома. Икс радостно начинает бегать туда-сюда между домом и своим хозяином с постоянной скоростью 3 мили в час. Икс не тратит время зря, поворачиваясь. К тому времени, как мистер Физ доберется до дома, сколько миль пробежит Икс?

    1. 3,5 мили
    2. 4,0 мили
    3. 4,5 мили
    4. 3,333… миль
    5. 3,555…
    6. миль
    7. Ни один из этих

    Ответ: (C) 4,5 мили

    Некоторые учащиеся могут попытаться составить ряд суммирования расстояний, пройденных собакой за время поездок между домом и хозяином. Это становится очень сложным.

    Простое решение: мистеру Физу нужно полтора часа, чтобы добраться до дома. Следовательно, собака бегает уже полтора часа. Со скоростью 3 мили в час собака прошла расстояние (3 мили в час) x (1½ часа) = 4,5 мили.
     

  8. Человек едет из города А в город Б со скоростью 40 миль в час и возвращается со скоростью 60 миль в час. Какова его средняя скорость в пути?

    1. 100 миль/ч
    2. 50 миль в час
    3. 48 миль в час
    4. 10 миль в час
    5. Ни один из этих

    Ответ: (C) 48 миль в час

    Ответ не зависит от расстояния между городами A и B. Предположим, что расстояние равно x, а расстояние туда и обратно равно 2x. Время пути из А в В равно х/40 ч, а время обратного пути х/60 ч. Для скоростей 40 миль в час и 60 миль в час время в оба конца составляет 2x/40 ч и 2x/60 ч. Средняя скорость определяется как V avg = общее расстояние ÷ общее время.

    Это становится упражнением в арифметике дробей. Ответ оказывается равным 48 милям в час, независимо от x. Студенты, скорее всего, перескочат к ответу 50 миль в час, так как это среднее значение заданных скоростей. Однако это не то, как определяется средняя скорость!
     

  9. Два поезда движутся навстречу друг другу со скоростью 17 миль в час и 43 мили в час. На каком расстоянии они будут друг от друга за 1 минуту?

    1. 60 миль
    2. 30 миль
    3. 6 миль
    4. 3 мили
    5. 2 мили
    6. 1 миля

    Ответ: (F) 1 миля

    Нет необходимости выполнять утомительные вычисления, если мы поймем, что каждый поезд приближается к другому с относительной скоростью (17 миль в час + 43 мили в час) = 60 миль в час = 1 миля/мин. Следовательно, за 1 минуту до столкновения они находятся на расстоянии 1 км друг от друга.

  10. Два шарика катятся по двум горизонтальным дорожкам. На одной дорожке есть провал, а на другой — бугор такой же формы. Какой мрамор победит?

    Ответ: На прямых участках пути два шарика имеют одинаковую скорость. Однако в каждой точке падения шарик имеет большую скорость, чем другой шарик в соответствующей точке горба. Таким образом побеждает шарик на дорожке с провалом. Этот аргумент предполагает, что шарики всегда остаются в контакте с дорожками.

  11. Три снаряда выпущены из одной точки над ровной поверхностью со скоростями V A , V B и V C . Все они достигают одинаковой максимальной высоты. Что из следующего верно относительно времени их полета?

    1. т А = т Б = т С
    2. т А > т В > т С
    3. т А < т В < т С
    4. Ни один из этих

    Ответ: (A) t A = t B = t C

    : Три снаряда имеют одинаковую максимальную высоту, следовательно, они имеют равные начальные вертикальные компоненты для своих скоростей. Таким образом, все они занимают одинаковое время, чтобы достичь максимальной высоты и вернуться на землю. (Учащиеся могут подумать, что, поскольку снаряды проходят разное расстояние по траектории, у них разное время в пути.)
     

  12. Три снаряда запускаются из одной точки над ровной поверхностью со скоростями V A , V B и V С . Все они достигают одинаковой максимальной высоты. Что из следующего верно относительно их начальной скорости?

    1. В А = В В = В С
    2. В А > В В > В С
    3. В А < В В < В С
    4. Ни один из этих

    Ответ: (C) V A < V B < V C

    Три снаряда имеют одинаковую начальную скорость и одинаковое время полета. Однако для их горизонтальных диапазонов X А < Х В < Х С . Горизонтальный диапазон обусловлен горизонтальными составляющими их скоростей в одно и то же время. Отсюда V Ax < V Bx < V Cx . Отсюда следует, что V A < V B < V C .
     

  13. Мяч несколько раз брошен с одной и той же высоты с одной и той же скоростью V o , но в разных направлениях A, B и C, как показано ниже. Он достигает земли со скоростью V A , V B и V C соответственно. Что из следующего верно относительно этих скоростей?

    1. В А = В В = В С
    2. В А > В В > В С
    3. В А < В В < В С
    4. Ни один из этих

    Ответ: (А) В А = В В = В С

    В каждом случае мяч стартует с одинаковой скоростью и, следовательно, с одинаковой кинетической энергией. Когда мяч ударяется о землю, он теряет такое же количество гравитационной потенциальной энергии и, следовательно, получает такое же количество кинетической энергии. Таким образом, в каждом случае мяч падает на землю с одинаковой скоростью.
    Здесь учащиеся могут подумать, что направление начальной скорости может влиять на скорость удара о землю.
     

  14. Барабан вращается с постоянной скоростью с вертикальной осью. Капля воды в точке Р на его поверхности отрывается и улетает. Глядя сверху, каков наиболее вероятный путь падения?

    Ответ: Капля воды P первоначально движется равномерно по окружности. Следовательно, в любой момент его скорость тангенциальна поверхности барабана. Когда он отделяется от барабана, больше нет центростремительной силы, заставляющей его двигаться по круговой траектории, поэтому он движется по касательной от поверхности.

    На трехмерном изображении капля будет падать на землю по параболе.
     

  15. Вес закрытой банки равен W, пока мухи внутри нее летают. Каков будет вес банки, если мухи поселятся внутри банки?

    1. Равен W
    2. Менее Вт
    3. Менее Вт

    Ответ: (A) Равно W

    Когда мухи летают, они давят на воздух, который, в свою очередь, давит на банку. На самом деле банка поддерживает мух, даже когда они летают. Если бы банку поместили на чувствительные весы, показания колебались бы около W и усреднялись бы на W в течение длительного интервала времени.
     

  16. Взвешивают закрытый сосуд с газом. Вносят ли молекулы газа вклад в измеряемый вес?

    1. Да, полностью
    2. Да, но частично

    Ответ: (A) Да, полностью

    Может показаться, что это похоже на вопрос 15. Однако в данном случае мы рассматриваем вопрос о том, влияет ли вес самого газа на вес всей системы. Один из подходов к этой проблеме состоит в рассмотрении вертикальных скоростей молекул. Когда молекула движется вниз, ее скорость увеличивается из-за ускорения под действием силы тяжести. Когда молекула сталкивается с дном сосуда, она передает силу, превышающую ее вес; избыточная сила как раз компенсирует время, в течение которого молекула не контактировала с банкой.

    Опять же, в микроскопическом масштабе, если банку поставить на чувствительные весы, показания весов будут варьироваться в районе W, но в среднем будут составлять W в течение достаточно длительного интервала времени.
     

  17. Как космонавты взвешиваются в состоянии невесомости?

    Ответ: Вес космонавтов на близкой орбите вокруг Земли составляет около 90 процентов от их веса на поверхности Земли. Однако они чувствуют себя невесомыми, потому что фактически находятся в свободном падении и, следовательно, не имеют нормальной силы от поверхности, действующей на них. Нормальная сила дает людям ощущение своего веса. Если человек встает на весы, чтобы определить свой вес, весы считывают нормальную силу, которую они прикладывают для поддержки этого человека. Следовательно, весы покажут нулевой вес для космонавта, если он «стоит» на таких весах в спутнике. Однако космонавты могут найти свою массу (инерцию), используя тот факт, что период колебаний системы пружина-масса зависит от присоединенной к ней массы, а не от силы тяжести. Устройство, разработанное НАСА по этому принципу, называется устройством для измерения массы тела (BMMD).
     

  18. Человек, несущий чашку воды с плавающим льдом, входит в лифт. Если лифт ускорится вверх, лед будет

    1. Поплавок выше
    2. Раковина глубже
    3. Остаться на прежнем уровне

    Ответ: (C) Оставайтесь на том же уровне

    Система отсчета с ускорением вверх эквивалентна инерциальной системе отсчета с более высоким значением ускорения свободного падения, определяемым как g’ = g + a. Выталкивающая сила на льду обусловлена ​​давлением воды, которое пропорционально силе тяжести g’. Вес блока составляет мг’. Следовательно, и вес блока, и выталкивающая сила увеличиваются в один и тот же раз при ускорении лифта вверх (и уменьшаются в один и тот же раз при ускорении лифта вниз). Поэтому лед плавает на одном уровне, а уровень воды в чашке не меняется.
     

  19. Почему у вертолета рядом с хвостом второй винт?

    Ответ: Так как основной (горизонтальный) винт вращается в одну сторону, то остальная часть вертолета стремится вращаться в противоположную сторону по закону сохранения углового момента. Вращение основного корпуса вертолета может быть предотвращено другим винтом рядом с хвостовой частью вертолета.
     

  20. В комнате есть три переключателя A, B и C. Два из них — фиктивные выключатели, а третий — выключатель настольной лампы в другой комнате. Вы можете включать и выключать три переключателя по своему усмотрению. Затем вы входите в комнату с настольной лампой только один раз и можете сказать, какой из выключателей является правильным для лампы. Как вы можете это сделать?

    Ответ: Включите выключатель А. Оставьте на несколько минут. Выключите A и включите B. Подойдите к настольной лампе.

    Если лампа горит, это B.

    Если лампа выключена и колба теплая на ощупь, это А.

    Если лампа выключена и колба холодная на ощупь, это C.
     

  21. Мальчик несет металлический стержень PQ горизонтально на пикапе, едущем по прямой горизонтальной дороге. В стержне индуцируется ЭДС из-за магнитного поля Земли, что делает конец P положительным (+), а конец Q отрицательным (-). Концы стержня теперь соединены проволокой. В каком направлении будет течь индуцированный ток в стержне, если он есть?

    1. от P до Q
    2. Q-P
    3. Через стержень не будет течь ток.

    Ответ: (C) Через стержень не будет течь ток.

    Стержень и проволока образуют замкнутый контур. Магнитный поток через петлю не изменяется при прямолинейном движении грузовика. Следовательно, по закону Фарадея в петле нет ни ЭДС индукции, ни индукционного тока.

Вопросы критического мышления по физике. Часть 2

  1. Две одинаковые чашки P и Q содержат одинаковое количество горячего кофе одинаковой температуры. Теперь в чашку P добавляют холодные сливки. Через несколько минут такое же количество холодных сливок той же температуры добавляют в чашку Q. Сравните новые температуры T P и T Q кофе в чашке. две чашки.

    1. Т Р = Т О
    2. Т Р > Т В
    3. Т Р < Т В

    Ответ: (B) T P > T Q

    Это вопрос о теплопередаче и законе охлаждения Ньютона. Тело при более высокой температуре отдает тепло окружающей среде с большей скоростью. Поскольку чашка Q находилась при более высокой температуре в течение более длительного интервала времени, она потеряла больше тепла.
     

  2. Металлический стержень АВ согнут в показанную форму.

    Если стержень нагревается равномерно, пространство между концами будет

    1. Увеличение
    2. Уменьшение
    3. Оставайся прежним

    Ответ: (A) Увеличение

    При тепловом расширении объекта любой формы каждая частица удаляется от любой другой частицы. Если точки А и В сблизятся, расширение будет противоположным.

    Прямые участки, заканчивающиеся точками A и B, расширяются и заставляют точки A и B сближаться. Однако расширение нижнего отрезка прямой раздвигает точки A и B. Длина нижнего сегмента больше, чем общая длина двух верхних сегментов. Следовательно, чистый эффект заключается в том, что точки A и B отдаляются друг от друга.

  3. Контейнер разделен на две половины перегородкой с отверстием. Две половины содержат один и тот же газ, но при разных температурах. Какая половина содержит большее количество газа?

    1. Половинка при более высокой температуре
    2. Половина при более низкой температуре
    3. Половинки содержат одинаковое количество газа.

    Ответ: (B) Половина при более низкой температуре

    Ответ можно быстро найти, используя известное уравнение PV = nRT. Отверстие в перегородке обеспечивает одинаковое давление в обеих половинах. Отсюда и количество молей.

  4. Бутылка полностью заполнена водой, как показано на схеме ниже. Какие из показанных точек имеют одинаковое давление?

    1. Р 1 и Р 2
    2. Р 2 и Р 3
    3. Р 1 и Р 3
    4. P 1 , P 2 и P 3
    5. Ни один из этих

    Ответ: (B) P 2 и P 3

    Давление жидкости в точке пропорционально глубине точки под открытой поверхностью жидкости. Это верно, даже если открытая поверхность не находится вертикально над точкой, как в случае с точкой P 3 . Можно подумать, что P 1 и P 3 имеют одинаковое давление, так как оба они находятся на 10 см ниже поверхности жидкости. Это неверно, потому что поверхность воды непосредственно над P 3 не является открытой поверхностью.
     

  5. Два одинаковых стакана держат воду на одной высоте, но в одном из них плавает деревянный брусок. Какой стакан весит больше?

    1. А
    2. Б
    3. Ни

    Ответ: (С) Ни

    Количество воды в стакане B меньше, чем в стакане A из-за воды, вытесненной плавающим блоком. Однако, применяя принцип Архимеда, вес плавающего блока равен весу вытесненной им воды.
     

  6. Два одинаковых стакана держат воду на одной высоте, но в одном из них полностью погружен деревянный брусок, прикрепленный на дне веревкой. Какой стакан весит больше?

    1. А
    2. Б

    Ответ: (А)

    В стакане B вытесненный объем воды заменен деревянным бруском меньшей плотности. Следовательно, он меньше весит.
     

  7. Два одинаковых стакана держат воду на одной высоте, но в один из них погружен железный брусок. Какой стакан весит больше?

    1. А
    2. Б
    3. Ни

    Ответ: (Б)

    В стакане В объем вытесненной воды занимает железный блок большей плотности. Следовательно, он весит больше.
     

  8. Два одинаковых стакана держат воду на одной высоте, но в одном из них на веревке подвешен железный блок. Какой стакан весит больше?

    1. А
    2. Б
    3. Ни

    Ответ: (С) Ни

    В стакане B отсутствует вода, вытесненная частично погруженным железным блоком. Выталкивающая сила, действующая на брусок, равна весу вытесненной воды. Эта сила также действует на дно стакана как реактивная сила и точно компенсирует уменьшение веса из-за недостатка воды.

  9. Два одинаковых стакана держат воду на одной высоте, но в одном из них на веревке подвешен полностью погруженный в воду железный блок. Какой стакан весит больше?

    1. А
    2. Б
    3. Ни

    Ответ: (С) Ни

    В стакане B на железный блок действует выталкивающая сила, равная весу вытесненной воды. Выталкивающая сила действует водой вверх на блок и как реакция вниз на дно стакана, таким образом компенсируя вес вытесненной воды.
     

  10. Лодка в озере бросает якорь в озеро. Уровень озера будет

    1. Остаться прежним
    2. Подъем
    3. Осень

    Ответ (С) Осень

    Находясь в лодке, якорь вытесняет воду, равную собственному весу. Поскольку плотность якоря больше плотности воды, объем вытесненной воды больше объема якоря. Когда якорь бросают в воду, якорь вытесняет объем, равный его собственному объему. Следовательно, объем воды, вытесняемой при броске якоря в озеро, меньше, чем при броске якоря в лодке, и уровень озера падает.
     

  11.  

    По озеру плывет двухтонная лодка. Выталкивающая сила на лодке должна быть
     
    1. 2 тонны
    2. Более 2 тонн
    3. Менее 2 тонн
    4. В зависимости от плотности воды в озере

    Ответ: (А) 2 тонны

    Находясь в лодке, якорь вытесняет воду, равную собственному весу. Поскольку плотность якоря больше плотности воды, объем вытесненной воды больше объема якоря. При падении якоря в воду якорь вытесняет объем, равный его собственному объему. Следовательно, объем воды, вытесняемой при броске якоря в озеро, меньше, чем когда якорь находился в лодке. Таким образом уровень озера падает.
     

  12. Когда мы смотрим на себя в плоское зеркало, мы видим перестановку влево-вправо, но не перестановку вверх-вниз. Почему?

    Ответ: В плоском зеркале имеется инверсия по глубине, а инверсия влево-вправо отсутствует. Право оказывается правым, лево — левым, верх — верхом, а низ — низом. Однако для человека, смотрящего в зеркало, возникает иллюзия переворота влево-вправо, а не переворота вверх-вниз. Это связано с тем, что для того, чтобы другой человек оказался лицом к наблюдателю, другой человек всегда поворачивается вокруг вертикальной оси, вызывая реальную перестановку влево-вправо.
     

  13. Длина волны красного света в воде близка к синему, но красные знаки выхода кажутся пловцу красным, находящемуся внутри воды. Почему?

    Ответ: Когда свет входит в среду из другой среды, его длина волны и скорость меняются, а частота остается прежней. На длину волны во второй среде не влияет введение третьей среды между двумя средами. Таким образом, красный свет, попадающий в глаз непосредственно из воздуха, имеет внутри глаза ту же длину волны, что и красный свет, сначала попадающий в воду, а затем в глаз, поэтому красный свет кажется пловцу под водой красным.
     

  14. Наши глаза наиболее чувствительны к зеленовато-желтому свету, но сигналы опасности красные. Почему?

    Ответ: Красный свет может проникать через атмосферу, содержащую пыль, облака и туман, намного эффективнее, чем любой другой цвет. Свет в синей части видимого спектра в гораздо большей степени рассеивается атмосферой. Вот почему небо кажется голубым, а закатное и восходное небо красноватым.
     

  15. Почему рождение электрон-позитронных пар не может происходить в вакууме?

    Ответ: Парное образование – это создание пары электрон-позитрон фотоном гамма-излучения. В этом процессе фотон исчезает, а его энергия преобразуется в массу покоя пары электрон-позитрон и кинетическую энергию, которую они несут. Часть энергии фотона превращается в массу покоя электрон-позитронной пары. Таким образом, линейный импульс пары меньше импульса фотона; это нарушение закона сохранения импульса. Следовательно, рождение пар всегда происходит вблизи тяжелого ядра. Отдача ядра констатирует сохранение импульса.

Автор:

Хасан Фахруддин
Академия наук, математики и гуманитарных наук Индианы BSU
Манси, Индиана

Средняя скорость и средняя скорость

Прежде чем понять среднюю скорость и среднюю скорость, мы должны сначала понять разницу между расстоянием и перемещением. Скалярная величина «расстояние» представляет собой расстояние, пройденное объектом. Кратчайшее расстояние между двумя точками представлено смещением, которое является векторной величиной. Например, если частица движется по кругу, расстояние, пройденное за один оборот, равно длине окружности, а смещение равно нулю.

Давайте посмотрим на определения скорости и скорости.

Различие между средней скоростью и средней скоростью 

Чтобы узнать о средней скорости и средней скорости, во-первых, мы должны знать некоторые термины и их значения.

Пройденное расстояние — Пройденное расстояние, как понятно из названия, представляет собой общее расстояние, пройденное объектом.

Затраченное время — время, необходимое объекту для перемещения на заданное расстояние.

Смещение. Смещение — это кратчайшее расстояние между начальной точкой, в которой находился объект, и конечной точкой, в которой он оказался.

Скорость. Скорость — это расстояние, пройденное объектом в единицу времени. Скорость является скалярной величиной. Это означает, что у него нет определенного направления. Скорость относится к тому, насколько быстро движется объект, или, по сути, к скорости, с которой преодолевается расстояние.

Скорость. Скорость — это полное перемещение объекта в заданном направлении в единицу времени. Скорость является векторной величиной. Это означает, что он имеет определенное направление. Скорость относится к скорости перемещения объекта во времени. Представьте себе человека, который проходит некоторое расстояние, прежде чем вернуться в исходное положение. Поскольку скорость — это скорость смещения, это движение приводит к нулевой скорости. Если человек хочет максимизировать свою скорость, он должен максимизировать смещение от своего исходного положения. Поскольку скорость является векторной величиной, при ее оценке мы должны следить за направлением.

Основное различие между скоростью и скоростью состоит в том, что скорость не учитывает направление, так как это скалярная величина, а скорость зависит от пройденного расстояния, тогда как скорость является векторной величиной, учитывающей направление, а скорость зависит от смещение.

Средняя скорость — это отношение общего расстояния, пройденного объектом, к общему затраченному времени. Однако средняя скорость представляет собой изменение положения или смещения (∆x), деленное на интервалы времени (∆t), в течение которых происходит смещение.

Итак, какая разница в определении средней скорости и средней скорости? Являются ли они одинаковыми с точки зрения параметров, используемых в соответствующих формулах? Предположим, что оба термина передают одно и то же значение; тем не менее, имеют ли они одни и те же единицы и обладают ли количествами одной и той же природы?

Хорошо! Ответы на все вопросы есть на этой странице. Кроме того, мы поймем разницу в средней скорости и формуле средней скорости, а также проиллюстрируем примеры из реальной жизни.

Средняя скорость

Средняя скорость любого объекта равна общему расстоянию, пройденному этим объектом, деленному на общее время, затраченное на преодоление указанного расстояния. Средняя скорость объекта говорит вам о средней скорости, с которой он будет преодолевать расстояние; то есть объект имеет скорость 30 км/час, его положение будет изменяться в среднем на 30 км каждый час. Средняя скорость — это показатель, который представляет собой количество, деленное на время, затраченное на получение этого количества. Единицей скорости в СИ является метр в секунду.

Средняя скорость рассчитывается по формуле S = d/t, где S — средняя скорость, d — общее расстояние, t — общее время.

Средняя скорость

Средняя скорость объекта может быть определена как смещение относительно исходного положения, деленное на время. Другими словами, это скорость, с которой объект перемещается со временем. Как и средняя скорость, единицей СИ является метр в секунду. Можно также сказать, что средняя скорость представляет собой отношение полного смещения объекта к общему времени, в течение которого это действие должно произойти.

Направление средней скорости является направлением смещения. Даже если скорость объекта колеблется и его величина меняется, его направление все равно будет таким же, как и направление смещения. Величина средней скорости всегда либо меньше, либо равна средней скорости, потому что перемещение всегда меньше или равно пройденному расстоянию.

Средняя скорость рассчитывается по формуле V = D/t, где V — средняя скорость, D — полное перемещение, t — общее время.

Формула для средней скорости и средней скорости

vср = Δx/Δt

Вы заметили, что формулы для средней скорости и средней скорости одинаковы.

Единственная разница заключается в типе физической величины, т. е. скорости и скорости. Скорость — это скалярная величина, которая имеет только величину. Однако скорость является векторной величиной, которая имеет как величину, так и направление.

Теперь рассмотрим некоторые задачи на среднюю скорость:

Задачи:

1. Автомобиль проезжает расстояние 70 км за 2 часа. Какова средняя скорость?

Ответ: средняя скорость = расстояние/время

Следовательно, средняя скорость автомобиля 70 км/2 часа = 35 км/час.

2. Человек может ходить со скоростью 1,5 метра в секунду. Какое расстояние он пройдет за 4 минуты?

Ответ: средняя скорость = расстояние/время

Расстояние = средняя скорость (время)

= 1,5(4) (60) = 360 метров

3. Поезд движется по прямой с постоянной скоростью 60 км/ч на определенное расстояние d, а затем проходит еще одно расстояние, равное 2d в том же направлении. с постоянной скоростью 80 км/ч в том же направлении, что и раньше. а) Какова средняя скорость поезда на всем пути?

Решение: а) Время t1 для преодоления расстояния d со скоростью 60 км/ч определяется как t1 = d / 60

Время t2 для преодоления расстояния 2d со скоростью 80 км/ч определяется как t2 = 2d / 80

Средняя скорость = расстояние/время = (d + 2d) / (d/60) + (2d/80)

= 3d / (80d + 2d × 60)/(60 × 80)

= 3 d/ (200d/4800) = 3d (4800)/200d = 72 км/ч

4. Вычислить среднюю скорость человека за определенный промежуток времени, если он проходит 7 м за 4 с и 18 м за 6 с по оси x -ось?

Решение: Начальное расстояние, пройденное человеком, xi = 7 м,

Конечное пройденное расстояние, xf  = 18 м,

Начальный интервал времени ti = 4 с,

Конечный интервал времени tf = 6 с,

Средняя скорость v = xi − xf / ti − tf   = 18 − 7 / 6 − 4 = 11 / 2 = 5,5 м/с.

Из приведенного выше текста мы понимаем, что средняя скорость любого объекта равна общему расстоянию, пройденному этим объектом, деленному на общее время, затраченное на преодоление указанного расстояния.

Средняя скорость объекта говорит вам о средней скорости, с которой он будет преодолевать расстояние; то есть объект имеет скорость 30 км/час, его положение будет изменяться в среднем на 30 км каждый час. Средняя скорость — это показатель, который представляет собой количество, деленное на время, затраченное на получение этого количества. Единицей скорости в СИ является метр в секунду.

Средняя скорость рассчитывается по формуле S = d/t, где S — средняя скорость, d — общее расстояние, t — общее время.

Средняя скорость

Из приведенного выше текста мы понимаем, что среднюю скорость объекта можно определить как смещение относительно исходного положения, деленное на время.

Другими словами, это скорость, с которой объект перемещается со временем.

Например, средняя скорость в системе СИ равна метрам в секунду. Можно также сказать, что средняя скорость представляет собой отношение полного смещения объекта к общему времени, в течение которого это действие должно произойти.

Средняя скорость объекта может быть определена как смещение относительно исходного положения, деленное на время. Другими словами, это скорость, с которой объект перемещается со временем. Как и средняя скорость, единицей СИ является метр в секунду. Можно также сказать, что средняя скорость представляет собой отношение полного смещения объекта к общему времени, в течение которого это действие должно произойти.

Направление средней скорости является направлением смещения. Даже если скорость объекта колеблется и его величина меняется, его направление все равно будет таким же, как и направление смещения. Величина средней скорости всегда либо меньше, либо равна средней скорости, потому что перемещение всегда меньше или равно пройденному расстоянию.

Средняя скорость рассчитывается по формуле V = D/t, где V равно средней скорости, D равно полному перемещению и t равно общему времени.

Теперь давайте рассмотрим некоторые задачи на среднюю скорость.

Задачи:

1. Водитель грузовика проезжает 20 км по дороге за 5 минут. Затем он дает задний ход и проезжает 12 км по дороге за 3 минуты. Какова его средняя скорость?

Решение: v = D/t

v = (20 — 12)/(5 + 3)

= 8/8 = 1 км/мин

2. Человек проходит 10 км на восток за 2 часа, а затем 2,5 км на запад за 1 час. Вычислите полную среднюю скорость человека?

Решение: vср = D/t

= (10  — 2,5)/2 + 1

= 7,5/3

vср = 2,5 км/ч

человека, если он пройдет 7 м за 4 с и 18 м за 6 с по оси абсцисс?

Решение: Начальное расстояние, пройденное человеком, xi = 7 м,

Конечное пройденное расстояние, xf = 18 м,

Начальный интервал времени ti = 4 с,

Конечный интервал времени tf = 6 с,

Средняя скорость vav = xi − xf / ti − tf  

= 18 /(6 − 4) = 11/2 = 5,5 м /s

Различия и сходства между средней скоростью и средней скоростью

Сходства – Оба эти термина являются средними некоторой длины по времени. Единица СИ и другие стандартные единицы измерения как средней скорости, так и средней скорости одинаковы. Формула, используемая для вычисления средней скорости и средней скорости, практически одинакова, v = D/t, s = d/t, с той лишь небольшой разницей, что в первом случае нужно указать направление.

Различия — Средняя скорость является скаляром и не зависит от наличия или отсутствия направления, в то время как средняя скорость, являющаяся вектором, нуждается в направлении. Средняя скорость зависит от расстояния, то есть общей длины, пройденной при измерении, в то время как средняя скорость зависит от смещения, то есть прямого расстояния от исходного положения до конечного положения.

Задачи, связанные как со средней скоростью, так и со средней скоростью

1. Автомобиль проезжает по прямой дороге на восток 120 метров за 5 секунд, затем 60 метров на запад за 1 секунду. Определить среднюю скорость и среднюю скорость.

Решение:

Расстояние = 120 метров + 60 метров = 180 метров

Перемещение = 120 метров – 60 метров = 60 метров на восток.

Прошедшее время = 5 секунд + 1 секунда = 6 секунд.

Средняя скорость = расстояние / прошедшее время = 180 метров / 6 секунд = 30 метров в секунду.

Средняя скорость = перемещение / прошедшее время = 60 метров / 6 секунд = 10 метров в секунду.

2. Бегун бежит по прямоугольной дорожке длиной = 50 метров и шириной = 20 метров. Он дважды проходит по прямоугольной дорожке и, наконец, возвращается к исходной точке. Если общее время, затрачиваемое им на бег по дорожке, равно 100 секундам, определите среднюю скорость и среднюю скорость.

Решение:

Окружность прямоугольника, равная расстоянию, пройденному за один круг = 2(50 метров) + 2(20 метров) = 100 метров + 40 метров = 140 метров.

Когда бегун дважды оббегает прямоугольник = 2(140 метров) = 280 метров.

Расстояние = 280 метров

Перемещение = 0 метров. (Поскольку бегун вернулся в исходную точку)

Средняя скорость равна расстоянию/прошедшему времени = 280 метров/100 секунд = 2,8 метра/секунду.

Средняя скорость равна перемещению / прошедшему времени = 0/100 секунд = 0

3. Человек начинает идти из точки на круглом поле радиусом 0,5 км и через 1 час оказывается в той же точке, где он изначально начал.

а) Какова средняя скорость на всем пути, который он проехал? Какова средняя скорость этого человека для того же?

Решение: а) Если этот человек обходит круглое поле и возвращается в ту же точку, то он прошел расстояние, равное длине окружности.

Таким образом, средняя скорость, которую он проехал = Расстояние/время = время прохождения окружности = π (0,5) (2)/1 час = 3,14 км/час (приблизительно).

б) Если он ходит по кругу и возвращается в ту же точку, с которой начал движение по кругу, то изменение его положения равно нулю. Поскольку изменение его положения равно нулю, перемещение также равно нулю. Это означает, что средняя скорость также равна нулю.

Q9 Основной единицей измерения скорости является i кммин ii мммин iii кмч iv мс…

Перейти к

  • Упражнение
  • Питание растений
  • Питание животных
  • Волокно к ткани
  • Нагревать
  • Кислоты, основания и соли
  • Физические и химические изменения
  • Погода, климат и приспособления животных к климату
  • Ветры, бури и циклоны
  • Почвы
  • Дыхание в организмах
  • Транспорт у животных и растений
  • Размножение растений
  • Движение и время
  • Электрический ток и его эффекты
  • Легкий
  • Вода: драгоценный ресурс
  • Леса: наш спасательный круг
  • История сточных вод

Главная > Решения НЦЭРТ Класс 7 Наука > Глава 13. Движение и время > Упражнение > Вопрос 16

Вопрос 16 Упражнение

Q9) Основная единица измерения скорости:

(i) км/мин (ii) м/мин (iii) км/ч (iv) м/с

Ответ:

Решение:

(iv) м/с.

Единицей измерения расстояния является метр (м), а единицы времени — секунда (с).

Скорость = (Расстояние/Время)

Следовательно, основной единицей измерения скорости является м/с .

Связанные вопросы

**Классифицируйте следующие движения как прямолинейное, круговое или колебательное:****(i) Двиг…

Q1) Классифицируйте следующее как движение по **прямой линии, круговое или колебательное движение**: (i) T…

Q2) Что из перечисленного неверно? (i) Основная единица времени – секунда. (ii) Каждый объект…

**Что из следующего неверно?****(i) Основная единица времени – секунда.****(ii) Каждый объект…

Q3) Простой маятник совершает 20 колебаний за 32 с. Каков период движения маятника…

**Простой маятник совершает 20 колебаний за 32 с. Каков период движения маятника?…

Фейсбук WhatsApp

Копировать ссылку

Было ли это полезно?

Упражнения

Упражнения

Главы

Питание в растениях

Питание у животных

клетчатка для ткани

Heat

кислоты, основания и соли

Физические и химические изменения

Погода, климат и адаптация животных к климату.

Добавить комментарий

Ваш адрес email не будет опубликован. Обязательные поля помечены *